You are on page 1of 70

1.

Supracondylar #:

A supracondylar fracture is a fracture of the distal humerus just above the epicondyles. While
relatively rare in adults it is one of the most common fractures to occur in children and is often
associated with the development of serious complications. Presenting complaints: The child
presents with history of a falling on an outstretched hand followed by pain, swelling and inability
to move the affected elbow.

On examination: Unusual prominence of olecranon process but because it is a supracondylar


fracture, the three bony point relationship is maintained, as in a normal elbow.

Neurovascular complications
i. tear or entrapment of the brachial artery
ii. spasm of the artery and
iii. compression of the artery relieved by manipulation of the fracture
iv. compression of median nerve. Causing Pink and Pulseless hand in supra condylar
fracture. Thus there is loss of circulation of forearm, causing lack of reperfusion of
tissues resulting in tissue death causing compartment syndrome.

Therefore the complications of elbow dislocations include the following:


i. Posttraumatic periarticular calcification, which occurs in 3-5% of elbow injuries
ii. Myositis ossificans or calcific tendinitis
iii. Neurovascular injuries (8-21% of cases) — ulnar nerve injuries are most common,
followed by brachial artery injuries (5-13%)
iv. Osteochondral defects, intra-articular loose bodies, and avascular necrosis of the
capitellum
v. Instability

Secondary injury
Most commonly brachial artery injury, and if left untreated could lead to Volkmann's
contracture (permanent flexion contracture of the hand at the wrist, resulting in a claw-
like deformity of the hand and fingers).

Taken from http://en.wikipedia.org/wiki/Supracondylar_fracture

A. T – among the commonest fractures in children (Apley’s concise 3rd edition page
311)
B. F – can cause compartment syndrome
(http://emedicine.medscape.com/article/1269576-treatment)
C. F – Occur as a complication of fracture of the lateral condyle of the humerus,
which may lead to tardy ulnar nerve palsy.
(http://en.wikipedia.org/wiki/Cubitus_valgus)
D. F – Existence of collateral arteries (Netter 4th edition page 434)
E. T – distal fragment may be displaced and / or tilted either posteriorly / anteriorly
/ medially / laterally / rotated (Apley’s concise 3rd edition page 311)
2. Colle’s #:

A Colles' fracture, also Colles fracture, is a distal fracture of the radius in the forearm with
dorsal (posterior) displacement of the wrist and hand. The fracture is sometimes referred to
as a "dinner fork" or "bayonet" deformity due to the shape of the resultant forearm. For a
more detailed discussion see distal radius fracture.

The term Colles fracture is classically used to describe a fracture at the distal end of the
radius, at its cortico-cancellous junction. However, now the term tends to be used loosely to
describe any fracture of the distal radius, with or without involvement of the ulna, that has
dorsal displacement of the fracture fragments. Colles himself described it as a fracture that
“takes place at about an inch and a half (38mm) above the carpal extremity of the radius”
and “the carpus and the base of metacarpus appears to be thrown backward”.

The classic Colles fracture has the following characteristics:


i. Transverse fracture of the radius
ii. 1 inch (2.54 cm) proximal to the radio-carpal joint
iii. dorsal displacement and angulation

Taken from http://en.wikipedia.org/wiki/Colle's_fracture

Complications:
i. Circulatory impairment
ii. Nerve injury (median nerve in carpal tunnel)
iii. Malunion
iv. Tendon rupture (tear of EPL)
v. Joint stiffness
vi. Complex regional pain syndrome

Treatment:
i. Undisplaced fracture – splintage
ii. Displaced fracture – CMR under anesthesia
iii. Comminuted and unstable fracture – external fixation with K wire

Taken from Apley’s concise 3rd edition page 325

A. T – so-called ‘dinner for deformity’ (Apley’s concise 3rd edition page 324)
B. F – dorsal displacement of the distal fragment of the radius (Apley’s concise 3rd
edition page 324)
C. F – control manual reduction is sufficient if its displaced (Apley’s concise 3rd
edition page 324)
D. T – can cause stiffness if it the joint is not being used for movement (Apley’s
concise 3rd edition page 324)
E. F – Tear of extensor pollicis longus (EPL) a few weeks after fracture (Apley’s
concise 3rd edition page 325)

3. # of the femoral neck:

In general, these injuries occur in 2 distinct populations, (1) young, active individuals with
unaccustomed strenuous activity or changes in activity, such as runners or endurance athletes,
and (2) elderly individuals with osteoporosis.
Taken from http://emedicine.medscape.com/article/86659-overview

The Shenton line is an imaginary line drawn along the inferior border of the superior
pubic ramus(superior border of the obturator foramen) and along the inferomedial
border of the neck of femur. This line should be continuous and smooth.
Interruption of Shenton's line can indicate (in the correct clinical scenario)
i. developmental dysplasia of the hip (DDH)
ii. fractured neck of femur

taken from http://radiopaedia.org/articles/shenton-s-line-1

Garden classified this fracture into four types:


i. Type 1 is a stable fracture with impaction in valgus.
ii. Type 2 is complete but non-displaced.
iii. Type 3 is displaced (often rotated and angulated) with varus displacement but
still has some contact between the two fragments.
iv. Type 4 is completely displaced and there is no contact between the fracture
fragments.

Taken from http://en.wikipedia.org/wiki/Hip_fracture

Nonoperative management is reserved only for those with extremely high surgical risk
or demented nonambulators with minimal hip pain

Taken from http://www.orthopaedia.com/display/Main/Femoral+neck+fractures

Operative treatment is almost mandatory. Displaced fracture will not unite without
internal fixation. Impacted fracture can be left to unite, but there is always a risk that
they may become displaced, even while lying in bed, so fixation is safer.
Complications include AVN, non-union, OA, general (thromboembolism, bed sores)

Taken from (Apley’s concise 3rd edition page 363)

A. F – most commonly seen in elderly osteoporotic people (Apley’s concise 3rd


edition page 362)
B. T – interruption of the Shenton’s line may indicate # of femoral neck
(http://radiopaedia.org/articles/shenton-s-line-1
C. F – Garden stage II shows complete but undisplaced fracture (Apley’s concise 3rd
edition page 362)
D. T – operative treatment is almost mandatory (Apley’s concise 3rd edition page
363)
E. T - (Apley’s concise 3rd edition page 362)

4.
Compartment syndrome:
Causes include:
i. Bleeding
ii. Edema
iii. Inflammation
iv. Tight plaster cast
Clinical features of compartment syndrome:
i. Pain
ii. Paresthesia
iii. Pallor
iv. Paralysis
v. Pulselessness

Treatment:
i. Cast, bandage and dressings must be removed
ii. Limb should be nursed flat (elevating the limb causes further decrease in end capillary
pressure and aggravates muscle ischemia)
iii. Fasciotomy
iv. Debridement if there is necrosis

A. T – bleeding, edema, inflammation (Apley’s concise 3rd edition page 294)


B. T – presence of pulse does not exclude the diagnosis (Apley’s concise 3rd edition
page 295)
C. T – may be caused by swelling of a limb inside a tight plaster cast (Apley’s concise 3 rd
edition page 294)
D. F – remove cast, bandages or dressings – and the limb should be nursed flat –
fasciotomy (Apley’s concise 3rd edition page 295)
E. ??

5. Indication of internal fixation


Indications for ORIF of fractures
http://wiki.answers.com/Q/Indication_for_open_reduction_with_internal_fixation#ixzz1Hz
Hp6VFz

Absolute
i. Unable to obtain an adequate reduction
ii. Displaced intra-articular fractures
iii. Certain types of displaced epiphyseal fractures
iv. Major avulsion fractures where there is loss of function of a joint or muscle group
v. Non-unions
vi. Re- implantations of limbs or extremities

Relative
i. Delayed unions
ii. Multiple fractures to assist in care and general management
iii. Unable to maintain a reduction
iv. Pathological fractures
v. To assist in nursing care
vi. To reduce morbidity due to prolonged immobilisation
vii. For fractures in which closed methods are known to be ineffective

Questionable
i. Fractures accompanying nerve of vessel injury
ii. Open fractures
iii. Cosmetic considerations
iv. Economic considerations
A. T when fracture is unstable and prone to displace
(http://wiki.answers.com/Q/Indication_for_open_reduction_with_internal_fixation)
B. F open fracture with gross contamination F
(http://wiki.answers.com/Q/Indication_for_open_reduction_with_internal_fixation)
C. F polytrauma, when to minimize risk of acute respiratory distress syndrome
D. T pathological fracture, whereby bone disease may prevent healing
(http://wiki.answers.com/Q/Indication_for_open_reduction_with_internal_fixation)
E. F shortage hospital bed
(http://wiki.answers.com/Q/Indication_for_open_reduction_with_internal_fixation

6.
Causes of pathological fracture
A pathological fracture is one that occurs in abnormal bone, as a result of a normally
insignificant stress.
Possible causes include:
a. metastatic tumours:
 breast, lung, thyroid, kidney, prostate
b. generalised bone disease:
 osteogenesis imperfecta, postmenopausal osteoporosis, metabolic bone
disease, myelomatosis, polyostic fibrous dysplasia, Paget's disease
c. local benign conditions:
 chronic infection, solitary bone cyst, fibrous cortical defect, chondromyxoid
fibroma, aneurysmal bone cyst, chondroma, monostotic fibrous dysplasia
d. primary malignant tumours:
 chondrosarcoma, osteosarcoma, Ewing's tumour
http://www.gpnotebook.co.uk/simplepage.cfm?ID=859111436

A. F repetitive stress (repetitive stress is a cause for stress fracture) (http://medical-


dictionary.thefreedictionary.com/pathological+fracture)
B. T osteomyelitis (Chronic infection which is a local benign condition)
(http://www.wrongdiagnosis.com/p/pathological_fracture/causes.htm)
(http://www.gpnotebook.co.uk/simplepage.cfm?ID=859111436)
C. T bone cyst (a local benign condition)
(http://www.gpnotebook.co.uk/simplepage.cfm?ID=859111436)
D. T previous traumatic fracture (pathological fracture is due to pathologic bone-
weakening condition)
(http://www.wrongdiagnosis.com/p/pathological_fracture/causes.htm)
E. T secondary to bone tumour ( osteosarcoma, osteoblastoma, metastatic tumors)
(http://www.wrongdiagnosis.com/p/pathological_fracture/causes.htm)
(http://www.gpnotebook.co.uk/simplepage.cfm?ID=859111436)

7.
The common sites of fractures in patients with osteoporosis are :
A. Femoral neck (True) (typical fragility occurs in vertebral column, hip, ribs , and wrist)
http://en.wikipedia.org/wiki/Osteoporosis
http://library.med.utah.edu/WebPath/TUTORIAL/OSTEO/OSTEOPOR.html
B. Distal tibia (False)
C. Sacrum (False)
D. Distal radius (True)
E. Vertebrae (True)
8. Traumatic anterior dislocation of the shoulder :
A. Is caused by forced adduction and internal rotation (False) (by forced abduction and
external rotation of the soulder) (APLEY’S CONCISE SYSTEM OF ORTHOPAEDICS
AND FRACTURES, THIRD EDITION, page 306)
B. Causes the head of the humerus to end up just below the coracoids process (True) (
X-ray shows head of humerus lying below and medial to the socket) (APLEY’S
CONCISE SYSTEM OF ORTHOPAEDICS AND FRACTURES , THIRD EDITION, page 306)
C. Is less common occurred as compare to posterior dislocation (False) (humeral head
displacement is usually anterior, less often posterior) (APLEY’S CONCISE SYSTEM OF
ORTHOPAEDICS AND FRACTURES , THIRD EDITION, page 306)
D. Would result in axillary nerve injury as a complication (True) (The axillary nerve may
be injured) (APLEY’S CONCISE SYSTEM OF ORTHOPAEDICS AND FRACTURES , THIRD
EDITION, page 306)
E. Would likely to become recurrent if occurred in younger patient (True) (When
shoulder dislocation occurs in adolescents and children, it has the worst natural
history of any injury; the rate of recurrence in later years is at least 70%) (
http://emedicine.medscape.com/article/1262004-overview)
9. Regarding fractured bone healing:
A. The process starts subsequently from inflammation stage, reparative stage and
remodelling stage T (complete phases are, tissue destruction, inflammation, callus
formation, consolidation and remodelling) Apley’s pg 268.
B. Stabilization of the fracture ends is necessary for healing process to occur T (in order
for bone healing to occur, immobilization is of important factor)
C. Osteoprogenitor cells within the periosteum are mobilized T
D. Healing is always associated with callus formation F (there is also healing without
callus) Apley’s 268
E. Remodeling stage would take two weeks to complete the process F (months or
years) Apley’s pg 269

Fractured healing
-can be divided into healing with callus and without callus.
Healing with callus (secondary bone healing)
-the process varies according to types of bone involved and amount of movement at
fractured site. Consists 5 stages:
a. Tissue destruction and haematoma formation
-vessels are torn and heamatoma forms around and within the fractured leads to
deprivation of blood supply and dies for few mm thick.
b. Inflammation and cellular proliferation.
-within 8 hours of the fracture there is an acute inflammatory reaction with proliferation
of cells under periosteum and within the breached medullary canal. Fragments end are
surrounded by cellular tissue and later bridges fractured site. New vessels form.
c. Callus formation
-potentially chondrgenic and osteogenic. The thick cellular mass (proliferating cell) with
its surrounding immature bone and cartilage forming callus or splint on the periosteal
and endosteal surface. Process aided by inductive proteins (fibroblast, growth factor,
transform GF and bone morphogenic protein)
d. Consolidation
-woven bone (immature bone) transformed into lamella bone. Fractured line filled by
osteoblastic activity. Slow process, may need several months for bone to be able to carry
normal load.
e. Remodelling
-process of alternating bone resorption and formation (reshaped) especially in children.

Healing without callus (primary bone healing)


-usually occur if the fracture site is absolutely immobilized (rigid fixation) eg: fracture rigidly
immobilize by internal fixationthere is no need for callus formation, instead new bone
formation occur directly between the fragments gap. (Gaps invaded by new capillaries and
bone-forming cell growing in from the edgesosteogenesisdirectly forming lamellar
bone! Wider gaps filled by woven bone first (gap healing)
-compared to healing with callus proliferating cells are needed to form cellular mass
together with woven bone and immature cartilage forming callus. It also ensues mechanical
strength while bone end heals!
-however, this type of healing is dependent to the internal fixation for its integrity due to
absence of callus. The implant also diverts stress away from the bone (reduces weight bare
of bone), therefore it is no fully recover until metal prosthesis is removed.
The time factor
-The rate of repair varies. Often depends upon:
i. Types of bone involvedcancellous bone heals faster than cortical bone. (Highly
vascularised)
ii. Types of fracturetransverse fracture heals longer than a spiral fracture
iii. State of blood supplypoor circulation means poor healing
iv. Pt’s general constituentshealthy bone heals faster
v. Pt’s agehealing is almost 2x faster in children than in adults.

Average time for fracture healing

Callus visible UL (2-3 weeks) LL (2-3 weeks)

Union UL (4-6 weeks) LL (8-12 weeks)

Consolidation UL (6-8 weeks) LL (12-16 weeks)

10. Factor associate in non-union fracture (answers directly from Apley’s pg 270)
A. Infection T
B. Smoking F
C. Splintage with POP F (this is to promote union)
D. Interposition of periosteum between fragments T
E. Excessive traction T

Delayed union, non union, and malunion.


Delayed union
-the time of treatment is prolonged (callus seen after expected time of treatment). Causes
can be due to either biological or biomechanical.
i. Poor blood supply
-often in badly displaced fracturecausing tearing of periosteum and interruption
of intramedullary BSfracture surface may become necroticdelayed callus
formation and delayed healing.
ii. Severe soft-tissue damage
iii. periosteal stripping
iv. Imperfect spilntage
-excessive traction (creating a fracture gap) and excessive movement at fracture site
will delay ossification in callus
v. Over rigid fixation
-rigid fixation delays rather than promote union.
vi. Infection
-tissue healing is severely hampered by bone lysis, necrotic and pus formation later
causing implant to loosen and fracture stability is lost.

-clinical features persistant fracture tenderness. More acute pain when subjected to
stress.

-treatment 2 important principles (1)to eliminate any possible cause of delayed union
(2)to promote healing by providing most appropriate biological env.
also, immobilization (cast or int fixation) to prevent movement at fracture site.
Still fracture loading is an important stimulus for union, so encourage muscular
exercise ang weight bare in cast or brace (partial weight bare).
if union is delayed >6 months and there is no signs of callus formation, int fixation
and bone grafting is indicated. (Operative)
Non-union
-can occur in either hyperthropic non-union or atrophic non-union.
-hyperthrophic non-unionbone end are enlarged, suggesting
osteogenesis still active but not capable of bridging the gap.
-atrophic non-unionthe bone end is tapered or rounded with no
suggestion of new bone formation.
-treatmentif symptomless, no need! Even if symptoms persist,
use props to stimulate union (eg: pulsed electromagnetic fields and
low frequency pulsed U/S)
(operative) hyperthrophic non-union, rigid fixation
may lead to union. Atrophic non-union, sclerosed bone
end should be excised and bone graft together with
rigid fixation should be done.
Malunion
-bone fragment join in an unsatisfactory position (false angulation, rotation and shortening)
-causes include (1)failure to reduce fracture adequately, (2)failure to hold reduction while
healing process, (3)gradual collapse of comminuted or osteoporotic bone.
-clinical featureobvious deformity esp limbs if compared to normal one. Sometimes, only
apparent on Xray.
-treatmentfew guidelines are offered:
i. In adult, angulation >10-15 degrees in a long bone or noticeable rotational deformity
may need correction by remanipulation or by osteotomy and int fixation.
ii. In lower limb, shortening of >2cm is acceptable, in case of severe limb shortening,
limb lengthening should be considered.
iii. Patient expectationcosmesis purpose.
iv. Angular deformity >15 degree in weight bearing joint to prevent OA.
v. In young children, angular deformity near the bone end will often remodel with
time. However, rotational not.
11.
Sports injuries of knee
A. Meniscus tear is most common F (ligamental tear is most common when it comes to
sport. Meniscus tear usually occur in young footballer due to weight bear on flex
knee with twisting force also in middle age due to fibrosis) Apley’s pg 226
B. Haemarthrosis usually occur due to torn meniscus T (meniscus tear may present
with joint effusion) Apley’s pg 227
C. Swelling of the knee for the next day commonly due to cruciate ligament tear F
(meniscus tear, usually swelling appear some hours later or the next day as
compared to ligamental tear, it appear immediately) Apleys pg 227 and374
D. Positive posterior drawer test indicate anterior cruciate ligament tear F ( when
positive ant drawer test, PCL is torn. When positive anterior drawer test, ACL is torn)
PE orthopaedic surgery, pg 132
E. Lateral meniscus tear is more common rather than medial meniscus tear F (medial is
more common as its attachement to the capsule make it less mobile) Apley’s 227

Meniscal tear
-menisci have an important role in (1)increasing the stability of the knee (2)controlling the
complex rolling and gliding actions of the joint (3)distributing load during movement.
-tear common in young adult (footballer) mechanism: weight on the flex knee together with
twisting strain. In middle life, tear occur with relatively little force due to fibrosis.
-medial meniscus is commonly affected due to its attachment to the capsule, male it less
mobile.
-patterns of tear:
i. Bucket-handle teartrauma (young
patient)
ii. horizontal tear
(transverse/radial)degenerative or
repetitive minor trauma.

-clinical features: pain is severe and further activity is


avoided, often the knee is locked in partial flexion.
Swelling appear hours later or following day. Sx
subside when rest. May occur periodically after strains or twist.
-in pt >40 age, may present with recurrent ‘’locking’’ and ‘’giving way’’ sx. Locking (inability
to extend the knee fully) suggest a Bucket-handle tearsometimes, they even learns to
unlock their knee.
-o/e, joint maybe slightly flex, tenderness over the joint line (commonly medial side), full
flexion but limited extension. Apley’s grinding test maybe positive!
-investigation MRI and athroscopy
-treatment arthroscopic surgery, cleanly excised displaced portion and post-op
physiotherapy.

12.
Safety and road traffic accident : (aku x jumpe explaination utk soklan ne, seems mcm kne
pikir logic je laa kot.)
A. Accident is misnomer term, because mostly accident caused by the negligence T
B. Usage of technologies can cause the accident T
C. Strict law totally abolish road traffic accident (RTA) F
D. Accident can give impact to economic growth and insurance T
E. Road design itself can induce accident T
13. TB spondylitis

i. Biopsy is a helpful diagnosis


ii. Multiple drug regime is the treatment of choice for uncomplicated case
iii. Destructive process caused by delayed hypersensitivity reaction

It is often difficult to distinguish TB from other types of infection or metastatic (klu based on
clinical features and spine x-rays only). If there is doubt, a needle biopsy may provide the
answer.
Other investigations may help in diagnosis of TB spondylitis. For example:
i. Mantoux test (positive)
ii. ESR (raised)
iii. Pus bacteriology examination and culture

A. TRUE
(Ref: Apley’s Concise System of Orthopedics and Fracture, 3rd edition, page 194)

For less advanced cases (no progressive bone destruction present), conservative treament is
usually sufficient and curative. Anti-tuberculous chemotherapy should be continue for 6-12
months. Anti-tuberculous chemotherapy are:
i. Rifampicin
ii. Isoniazid
iii. Pyrazinamide
iv. Ethambutol
However, there are some criterias/indications for operation:
i. Abscess formation (must be drained)
ii. Marked bone destruction and progressive deformity (requires spinal fusion)
iii. Threatened paraplegia that does not respond to conservative treatment

B. TRUE
(Ref: Apley’s Concise System of Orthopedics and Fracture, 3rd edition, page 194 and 195)

Delayed hypersensitivity reactions occurs 48 to 72 hours after antigen exposure. It is a major


mechanism of defense against various intracellular pathogens, including mycobacteria, fungi
and certain parasites, and it occurs in transplant rejection and tumour immunity. The host
respone againts intracellular pathogens such as Mycobacterium tuberculosis is markedly
impaired due to loss of CD4+. This lead to granulomatous formation, which contain enzyme
that caused bone destruction.

C. TRUE
(Ref: http//emedicine.medscape.com/article/136118-overiew)

14. Spinal injury:


i. Vertebral fracture commonly cause spinal cord injury
ii. Neurovascular examination is performed to reveal the level of spinal cord injury
iii. Resuscitation is the initial management of spinal cord injury
Vertebral fracture and spinal cord injuries are due to:
i. direct force eg: penetrating wounds from firearms @ knives)
ii. indirect force (more common) eg: following a fall from a height when the spinal
column collapses in its vertical axis, during violent free movements of the neck or
trunk

There are mechanisms come into play, often simultaneously that lead to spinal cord injury:
i. axial copmression
ii. flexion
iii. extension
iv. rotation
v. shear
vi. distraction

Ade beberape types of vertebral fractures yg patut kita ambik perhatian bcoz ade fracture
of spine yg common menyebabkan spinal injury dan ade jgk yg extremelly rare
menyebabkan spinal injury:

i. Wedge fracture (flexion – compression mechanism) . Neurological involvement -


extremelly rare
ii. Burst fracture (axial – compression mechanism) . Neurological involvement –
common due to retropulsion
iii. Flexion – dislocation mechanism. Neurological involvement – common

So, as a general vertebral fracture commonly cause spinal cord injury.


A. TRUE
(Ref: Apley’s Concise System of Orthopedics and Fracture, 3rd edition, page 342 and 349)

Hangman’s fracture

i. Known as traumatic spondylolisthesis of c2


ii. First noted on crimal in death sentenced by hanging as post-mortem examination
iii. Symptom – pain radiating along the course of greater occipital nerve
iv. Initial management – immobilize with Philadelphia collar

Spondylolisthesis means vertebral displacement. Listhesis is nearly always between L4 and


L5 or between L5 and the sacrum. It can be classifified into:
i. Dysplastic spondylolisthesis
ii. Lytic spondylolisthesis
iii. Degenarative spondylolisthesis

However, in some scholars they classified spondylolisthesis into 6 group:


i. Dysplastic spondylolisthesis
ii. Isthmic spondylolisthesis
iii. Degenerative spondylolisthesis
iv. Traumatic spondylolisthesis
v. Pathologic spondylolisthesis
vi. Iatrogenic spondylolisthesis
15. In Hangman’s fracture, the pedicles of the axis (C2) are fractured and the C1/2 disc is torn;
the mechanism is extension with distraction. It can presented with undisplaced or displaced
fracture.
Traumatic spondylolisthesis is extremely rare, results from a traumatically-induced fracture
to the neural arch other than the pars region. The most common example of traumatic
spondylolisthesis is Hangman’s Fracture.

A. So the answer for (B) is TRUE


(Ref: Apley’s Concise System of Orthopedics and Fracture, 3rd edition, page 199 and
345, http: //www.chirogeek.com/005_Spondylolisthesis_main_final.htm# Traumatic)

B. TRUE

I am sorry.I could not find the answer. Tp kt cni kite nk share clinical features of
Hangman’s fracture as general. The presentation may be late. Initial symptoms are
often slight and the patient usually experiences occipital neuralgia with some local
discomfort and stiffness of the upper cervical spine. Inded, the symptoms and signs
may resemble miningism.

(Ref: http://web.jbjs.org.uk/cgi/reprint/57-B/1/82.pdf)

C. The management of Hangman’s fracture is based on types of fracture. For


undisplaced fractures, they are threated in semi rigid collar (Philadelphia cervical
collar) or halo-vest until united. Displaced fractures may need reduction before
immobilization in a halo-vest for 12 weeks.

D. TRUE

(Ref: Apley’s Concise System of Orthopedics and Fracture, 3rd edition, page 345,
http://journals/lww.com/euro-
emergencymed/Fulltext/2001/03000/Indications_of_Philadelphia_collar_in_the 7.aspx)

16.
In low back pain

Symptoms of cauda equina syndrome are:


i. Low back pain
ii. Pain in leg (unilateral or bilateral) that starts in the buttocks and travels down the
back of the thighs and legs (sciatica)
iii. Numbness in the groin or area of contact if sitting on the saddle (perineal or saddle
paresthesia)
iv. Bowel and bladder disturbances (urinary and fecal retention)
v. Lower extremity muscle weakness and loss of sensations
vi. Reduced or absent lower extremity reflexes
vii. Impairment of penile erection

A. TRUE
(Ref: Apley’s Concise System of Orthopedics and Fracture, 3rd edition, page 98,
http://www.emedicinehealth.com/cauda_equina_syndrome/page2_em.htm)

The commenest age to develop a prolapsed disc is between 30 and 50 years old. Twice as many
men as women are affected

B. TRUE
(Ref: http://www.patient.co.uk/health/Prolapsed -Disc-(Slipped-Disc).htm)

Since most episodes of lower back pain are self-limited, it is often advisable for patients to
employ back care on their own early in the course of low back pain.

In most cases, do-it-yourself back care for low back pain should center on a combination of:
i. A short course of rest, limited to one to two days
ii. Pain medication, such as NSAIDs (e.g. ibuprofen) and/or acetaminophen
iii. Application of ice and/or heat on the lower back to decrease inflammation.

Slow mobilization and gentle stretching is then an advisable form of lower back pain care, and
the sooner a patient can return to his or her normal functional activities, the sooner the episode
of lower back pain will usually get better.

Other form of lower back pain are:

i. Walking is often an excellent exercise for low back pain since it is gentle on the
back and helps oxygenate the soft tissues in the back to stimulate a healing
response.

ii. If walking is too painful, exercising in the water (water therapy or pool therapy)
is usually tolerable. Such back care is typically beneficial for lower back pain
because the water counteracts gravity and helps to support the patient’s weight
in a controlled fashion.

iii. Sitting upright (e.g. in an office chair, driving) will often aggravate low back pain,
since this position loads the back three times more than standing. Sitting in a
reclining position, however, relieves pressure on the lower back and is often the
most comfortable position for patients experiencing an episode of back pain in
the lower back (lumbar spine).

C. TRUE
(Ref: http://www.spine-health.com/conditions/lower-back-pain/back-care-lower-back-pain

The answer will be TRUE if the question is more specific. As I mentioned early, according to
Apley’s there have 3 types of spondylolisthesis. Spinal claudication only occur in
degenerative type of spondylolisthesis due to narrowing of the spinal canal; not the typical
presentation for all types of spondyllolisthesis.
D. FALSE
(Ref: Apley’s Concise System of Orthopedics and Fracture, 3rd edition, page 198)
Clinical symptoms pun dah cukup utk diagnose acute disc prolapsed (ADP) which are sudden
back pain with nerve root symptoms. In most cases, no test are needed as the symptoms
often settle within a few weeks. Some people do not have symptoms sebab prolapse tu kecil
or occur away from the nerves. Spine X-rays or scan may be advised if symptoms persist.
MRI scan dpt bg more information regarding the site and size of a prolapsed disc. MRI dpt
highlightkan soft tissue. So, from x-ray tak byk info yang kita boleh dpt tentang abnormality
in ADP.

E. TRUE
(Ref: http://www.patient.co.uk/health/Prolapsed -Disc-(Slipped-Disc).htm)

17. Loose bodies in the knee joint can be produced by


A. Synovial chondromatosis T
B. Osteochondritis dissecans T
C. Osteoarthritis T
D. Pigmented villous nodular synovitis T
E. Osteochondral fracture T

a. Regarding loose bodies in the knee


- Patients complain of sudden locking of the joint which is usually reversible
- A pedunculated loose body may be felt on palpation; one that is truly loose tends to
slip away ‘joint mouse’
- X-ray: most loose bodies are radioopaque, and the film may show an underlying joint
abnormality
- Rx: a loose body in the joint causing symptoms should be removed with the aid of
arthroscopy unless the joint is severely osteoarthritic
- other causes of loose bodies in the knee
o Charcot’s disease – large osteocartilaginous bodies are separated by repeated
trauma in a joint that has lost protective sensation
References: Apley’s Concise System of Orthopaedics & Fractures 3rd edition, page
230

b. Synovial chondromatosis
- Synovial chondromatosis is a rare and benign metaplasia of the synovial
membrane resulting in the formation of multiple intra-articular cartilaginous
bodies, sometimes HUNDREDS of loose bodies
- Causes pain and limitation in mobility
- Most often in middle aged men
- Location:
i. over one-half of cases occur in the knee, followed by the elbow
ii. other common sites include the hip, shoulder, wrist and ankle
iii. when located in the foot or ankle the term "soft tissue chondroma" may be
used

c. Classification:
i. early: no loose bodies but active synovial disease
ii. transitional: active synovial disease, and loose bodies
iii. late: loose bodies but no synovial disease
d. On x-ray may show intraaricular loose bodies, if not seen then do MRI (T2 weighted)
e. Rx: total open synovectomy treatment of choice
A. True
References:http://www.wheelessonline.com/ortho/synovial_chondromatosis,
http://www.bonetumor.org/tumors-unknown-type/synovial-chondromatosis, Apley’s
Concise System of Orthopaedics & Fractures 3rd edition, page 230

B. TRUE Osteochondritis dissecans

- Osteochondritis is a group of conditions where there is compression (in crushing),


fragmentation (in traction) or separation (in dissecansdissect) of a small segment
of bone, usually at the bone end and involving the attached articular surface
- It is a condition where bone damage and necrosis follows trauma to articular
surfaces
- The affected portion of bone shows many features of ischaemic necrosis, including
increased vascularity and reactive sclerosis in the surrounding bone on x-ray
- Mostly occurs in children & adolescents
- It can be divided into 3 types:
i. Shearing osteochondritis (osteochondritis dissecans):
- In which a small, well-demarcated piece of bone and overlying
cartilage becomes separated and forms a loose body because of
avascularity
- May form 1 or 2 loose bodies
- Most commonly is due to repeated minor trauma producing an
osteochondral fracture of a convex joint surface
- Most commonly affects the knee; especially the lateral part of the
medial femoral condyle
- On x-ray the dissecting fragment is defined by a radiolucent line of
demarcation and when it separates a crater is obvious
- Rx:
a. Fragment in position – conservative: weight relief & restrict
activity
b. Fragment detached + symptoms – operative: fix back in
position or remove completely
ii. Crushing osteochondritis:
- The ossific nucleus which undergoes avascular necrosis, is crushed
under pressure
- Spontaneous; no cause
- Areas usually affected:
o Metatarsal head – Freiberg’s disease
o Navicular bone – Kohler’s disease
o Lunate bone – Kienbock’s disease
o Capitulum – Panner’s disease
- On x-ray shows increased density, in later stages bone collapse can
be seen
- Rx:
a. Conservative: analgesics + splintage
b. Lunate affected - operative
iii. Pulling osteochondritis (traction apophysitis)
- Excessive pull by a large tendon may damage the unfused epiphysis
to which it is attached
- Typically occurs at 2 sites:
o Tibial tuberosity - Osgood-Schlatter’s disease
o Calcaneal apophysis – Sever’s disease
- On x-ray shows increased density due to slight trauma rather than
necrosis
- Rx:
a. Conservative: rest
References: Apley’s Concise System of Orthopaedics & Fractures 3rd edition, page 228-230

C. TRUE Osteoarthritis
- Osteoarthritis is a chronic joint disorder in which there is progressive
softening and disintegration of articular cartilage accompanied by new
growth of cartilage and bone at the joint margins (osteophytes) and capsular
fibrosis
- 2 mechanisms:
- weakening of articular cartilage (due to genetic type II collagen defect or
enzymatic activity in inflammatory disorders such as RA)
- or increased mechanical stress in some parts of articular surface (due to
excessive impact loading or joint incongruity)
- or both
- Features
i. Early: insidious pain, stiffness which is worse after periods of rest
ii. Advanced: sweliing, deformity (usually genu varus if knees affected),
loss of mobility and muscle wasting
- No systemic manifestations (as opposed to RA)
- Pieces of cartilage or osteophyte can come loose and form loose bodies
- Can be primary (no obvious cause) or secondary (follows a joint disease or
injury)
- 3 characteristic features on x-ray:
i. Reduced joint space (due to cartilage depletion)
ii. Subarticular cyst formation and sclerosis
iii. Osteophyte formation
- Rx:
i. Early: conservative - pain relief, joint mobility, load reduction
ii. Intermediate: joint debridement (for knee) or realignment
osteotomy (for hip or knee)
iii. Late: surgery
a. Indications: unrelieved pain
b. Progressive disability
c. Types:
i. Arthroplasty (op of choice for >60 y/o)
ii. Arthrodesis (if stiffness can be tolerated because it
will eliminate movement)
References: Apley’s Concise System of Orthopaedics & Fractures 3rd edition, page 41, 230

D. TRUE Pigmented villous nodular synovitis


- It is a slow-growing, benign and locally invasive tumor which affects synovial
lined joints, bursae and tendon sheaths characterized by fibrous stroma,
hemosiderin deposition, histiocytic infiltrate and giant cells
- Villi formed in patients with PVNS can twist, infarct, and form a loose body
- Location: most often involves the knee (also in hip, ankle, elbow, etc.)
- always consider PVNS in a younger patient with unexplained hip pain
- Acute episodic attacks of pain and swelling may occur
- Patients may have mechanical symptoms (locking and catching)
- Most have hemorrhagic, dark brown synovial fluid
- Biopsy is diagnostic
- Sub-types: it usually presents as a monoarticular hemarthrosis, and may exist in
a nodular or a diffuse form
- diffuse form:
i. disease may be active or inactive
ii. look for peri-articular erosions on radiographs
iii. diffuse mass may be present on exam
- nodular form:
i. less common than the diffuse form of the disease
ii. does not show the same destructive changes as the diffuse form of
PVNS
iii. may cause recurrent hemarthrosis and aspirate may be of normal color
(may not show classic brown color)
- Rx:
i. arthroscopic synovectomy
o may be indicated for nodular form or for inactive form of diffuse disease
ii. open synovectomy
o treatment of choice for patients w/ active form of diffuse disease

References:http://books.google.com.my/books?id=TbxYM_Ts-
3YC&pg=PA83&lpg=PA83&dq=pigmented+villous+nodular+synovitis&source=bl&ots=K_wx0
nyZee&sig=Nigi-
lNiUyefw39vAaUjDPHHEM0&hl=en&ei=DVqLTeCfOovMrQeBlMTNDg&sa=X&oi=book_result
&ct=result&resnum=10&ved=0CGAQ6AEwCQ#v=onepage&q=pigmented%20villous%20nod
ular%20synovitis&f=false,
http://www.wheelessonline.com/ortho/pigmented_villonodular_synovitis

E. TRUE Osteochondral fracture (injury)


- An osteochondral fracture is a type of fracture in which the articular cartilage at the
end of a joint becomes torn
- These fractures are most commonly seen in the knee and ankle joints, as these joints
take a lot of strain and bear a lot of weight, which make them vulnerable to damage
- When an osteochondral fracture occurs, it is common for there to be fragments of
bone and cartilage inside the joint. Sometimes they remain attached to the joint, in
which case they are known as stable, while in other instances, they are unstable,
floating around inside the joint.
- These fragments are a cause of concern because they can grind at the joint, causing
additional damage in addition to making the joint rather painful

References: http://www.wisegeek.com/what-is-an-osteochondral-fracture.htm, Apley’s


Concise System of Orthopaedics & Fractures 3rd edition, page 23

18.
The features that should trigger more active investigation of TB of the joint
A. A long history of joint swelling T
B. Involvement of multiple joint T
C. Marked synovial thickening T
D. Periarticular osteoporosis T
E. Marked muscle swelling F
- TB can affect the vertebra and large synovial joints
- Features of TB of the joint include pain, muscle wasting, synovial thickening, limited
movement, stiffness, deformity. In late cases there may be a sinus
- Diagnosis of TB of the joint may not be suspected in areas where TB is not endemic
because in many respects it resembles rheumatoid arthritis
- Features that are suggestive of TB of the joint and which calls for more active
investigations include:
i. Long history
ii. Involvement of only 1 joint
iii. Marked synovial thickening
iv. Marked muscle wasting
v. Periarticular osteoporosis on x-ray
- ESR is usually raised and Mantoux test is +ve
- Synovial biopsy for histological examination and culture often necessary
Reference: Apley’s Concise System of Orthopaedics & Fractures 3rd edition, page 25

A. True
- Involvement of multiple joints
- TB of the joint is a chronic monoarthritis affecting a large joint, usually the hip or
knee
B. False
Marked synovial thickening
C. True
Periarticular osteoporosis on x-ray
D. True
Marked muscle swelling
Marked muscle wasting is characteristic in joint TB
E. False

19.
Factors/aetiology of DDD
A. Obesity T
B. Genetic and hereditary T
C. Frequent cracking the knuckles of finger F
D. DM T
E. Gout F

Regarding DDD:
- Degenerative disc disease is not really a disease but a term used to describe the
normal aging changes in spinal discs
- The discs act as shock absorbers for the spine, allowing it to flex, bend, and twist
- Degenerative disc disease can take place throughout the spine, but it most often
occurs in the discs in the lower back (lumbar region) and the neck (cervical region)
- The changes in the discs can result in back or neck pain as well as:
- Osteoarthritis
- Herniated disc
- Spinal stenosis
- These conditions may put pressure on the spinal cord and nerves, leading to pain
and possibly affecting nerve function
- As we age, our spinal discs break down, or degenerate, which may result in
degenerative disc disease in some people. These age-related changes include:
- Dehydration of discs due to reduced water attracting molecules. This reduces the
ability of the discs to act as shock absorbers and makes them less flexible. Loss of
fluid also makes the disc thinner and narrows the distance between the vertebrae
- Tiny tears or cracks in the outer layer (annulus fibrosus) of the disc due to changes in
collagen structure. The jellylike material inside the disc (nucleus pulposus) may be
forced out through the tears or cracks in the capsule, which causes the disc to bulge,
rupture, or break into fragments
- These changes are more likely to occur in people who:
i. smoke cigarettes because it will reduce the amount of water in discs
ii. do heavy physical work (such as repeated heavy lifting)
iii. people who are obese
o genetics – some people may inherit a prematurely aging spine
- An acute injury leading to a herniated disc (such as a fall) may also begin the
degeneration process
- As the space between the vertebrae gets smaller, there is less padding between
them, and the spine becomes less stable and more mobile
- The body reacts to this by constructing bone spurs (osteophytes) to reduce the
hypermobility
- Osteophytes can put pressure on the spinal nerve roots or spinal cord, resulting in
pain and affecting nerve function
- The pain often gets worse with movements such as bending over, reaching up, or
twisting (mechanical)
- Diagnosis is mainly clinical, but radiography (AP and lateral taken) may show signs of
degeneration such as loss of disk height, sclerosis of the endplates, or osteophytic
ridging In addition, spondylolisthesis can be diagnosed and the degree of slippage
visualized easily on lateral images. Oblique views may be helpful is spondylolysis is
suggested. CT and MRI may be more specific
- Rx: (for prolapse/herniation)
i. Conservative: pain relief, exercise
ii. Definitive: 4 R’s
a. rest
b. reduction
c. removal
 lumbar: operative  discectomy. Indications:
a. cauda equina compression syndrome
b. persistent pain and severely limited straight leg raising after 2
weeks conservative Rx
c. neurological deterioration
d. frequently recurring attacks
 cervical: operative  anterior disc removal and fusion, rarely
indicated
d. rehabilitate
References:http://www.webmd.com/back-pain/tc/degenerative-disc-disease-topic-
overview, http://www.spineuniverse.com/conditions/degenerative-disc/what-degenerative-
disc-disease, Apley’s Concise System of Orthopaedics & Fractures 3rd edt, page 182-183, 195-
197
A. True
Obesity
B. True
C. FALSE
Frequent cracking the knuckles of finger
Unrelated to the intervertebral discs
D. True
- Diabetes mellitus
- There may be an association between diabetes mellitus and development of DDD
according to a study done by Anekstein et al, although the percentage is not high
Reference: http://www.ncbi.nlm.nih.gov/pubmed/20450123
E. False
- Gout
- Gout usually affects the large joint of the big toe, but can also affect other joints,
such as the knee, ankle, foot, hand, wrist and elbow
- In rare cases, it may later affect the shoulders, hips or spine. Gout does not spread
from joint to joint
- Although gout can affect the spine, there is no evidence to say that it can lead to
DDD
References: http://www.healthcentral.com/osteoarthritis/h/can-gout-affect-in-your-arms-
hand-over-shoulders-and-back.html

20. Regarding amputation


A. Diabetic gangrene of the foot require amputation at the distal tibia F
B. Below knee amputation ,weight is taken on the stump F
C. Above knee amputation,weight is taken on the ischial tuberosity T
D. Elderly patient refuse to use above knee prostheses because of the high energy
requirement T
E. Pain due to neuroma formation is a complication T

Indications for amputation  3D’s


i. Dead eg in PVD (most common – 90%)
ii. Dangerous eg in malignant tumors, lethal sepsis, crush injuries
iii. Damned nuisance eg pain, gross malformations, recurrent sepsis or severe loss of
function
Reference: Apley’s Concise System of Orthopaedics & Fractures 3rd edition, page 131-134

A. False
- Diabetic gangrene of the foot require amputation at the distal tibia
- For diabetic gangrene there are 2 types of amputations: minor and major
- Minor (partial foot) amputations: eg Ray amputation where a toe and part of the
corresponding metatarsal bone is removed and the wound is usually left open to
heal, or transmetatarsal amputation
- Rarely major (wholefoot) amputations: eg Syme’s which is through-ankle
amputation

B. FALSE In below knee amputation, weight is taken on the stump


- The patient who underwent below the knee amputation usually uses the knee for
weight bearing rather than step on the stump when moving from place to place
without wearing the prosthesis
- Stepping on the stump may induce pain and pressure wound can develop on the
skin that covers the bone
- Protrusion of the bone through the skin is a very serious complication
- It is thus obvious that an amputee could not transfer the body weight on the bottom
of the stump
Reference: http://www.freepatentsonline.com/4778470.html

Weightbearing areas of amputations:


i. Transmetatarsal, Syme’s and knee disarticulation all bear weight on the stump
ii. Below knee amputations bear weight on the patella tendon, the lateral part of the
lower limb. There should be no pressure on the fibula head, tibial plate, hamstrings
or end of stump as this can lead to severe pressure sores
Reference: http://www.rehabsa.co.za/content/articles/Amputation.pdf

C. True
- In above knee amputation, weight is taken on the ischial tuberosity
- The above knee amputee will support their body weight on the ischial tuberosity
(seat bone), with the soft tissue of the residual limb bearing only a minimal amount
of weight
Reference: http://www.brownfieldstech.com/above_knee.asp

D. True
- Elderly patients refuse to use above knee prostheses because of the high energy
requirement
- As the amputation level rises so does the energy expenditure necessary to walk
- Individuals with amputations of the distal third of the foot (transmetatarsal level)
often achieve near normal mobility with the aid of a custom insole
- A below knee amputation (BKA) requires a 25 percent increase in energy
expenditure to ambulate (walk)
- Walking with an above knee amputation (AKA) requires 65 percent more energy
than the normal state
Reference: http://www.reversegangrene.com/A.htm

E. True
Pain due to neuroma formation is a complication
Complications of amputation
a. Early:
i. breakdown of skin flaps - due to ischemia or suturing under excessive
tension or an unduly long tibia pressing against the flap
ii. gas gangrene – due to clostridia and spores from perineum in high above
knee amputation
b. Late:
i. skin
- eczema or tender purulent lumps in the groin
- ulceration – due to poor circulation
ii. muscle - instability if too much muscle at end stump
iii. artery - cold, blue stump due to poor circulation which is liable to
ulcerate
iv. nerve - a cut nerve always forms a tiny ‘neuroma’ which is occasionally
painful
v. phantom limb
Reference: Apley’s Concise System of Orthopaedics & Fractures 3rd edition, page 131-134
Reference for question 21-24:Apley’s concise system pg116-117

21. Median nerve injury


A. Low lesions may be caused by cuts in front of the wrist or by carpal dislocations - t
B. The is thenar eminence wasted - t
C. Thumb abduction and opposition are weak - t
D. Sensory is lost over the one and a half fingers - f
E. Trophic changes may be seen - t

Anatomy
Forearm
Superficial flexor
i. Pronator teres
ii. Flexor caopi radialis
iii. Palmaris longus
iv. Flexor carpi ulnaris
Intermediate flexor
i. Flexor digitorus superficialis
Deep flexor
i. Flexor degitorum profundus
Supply by AIN (ant. interossious.n)
ii. Flexor pollicis longus
iii. Pronator quadrate
Sensory: NONE

Hands
Thenar.m (consist of: abductor pollicis, flexor digiti minimi, opponens digiti minimi digiti
minimi stand for little finger and it fn base on it name) and part of intrinsic.m- lat 2 of
lumbrical.

Sensory: thumb, index, middle and half of ring finger. – specific point at the tip of index
finger

Low lesion
Cause: cut at the wrist, carpal bone dislocation
a. Effect: wasting of the thenar area
i. Weak thumb abduction and opposition
ii. Loss of sensation over the lateral 3 and half of the finger at palmar surface
iii. Trophic change

High lesion
Cause: elbow dislocation and frac at the forearm
a. Effect: pts unable to flex the thumb, index and middle finger when we ask the pts to
genggam
i. jari  pointing index sign- bcoz lateral aspect of flexor digitorum profundus is
supply by
ii. the median.n. it fn is to flex thumb, index and middle finger.
iii. In high lesion (lesion at the elbow) flexor digitorum profundus is also be affected

iv. there is absent of flexion of the thumb, index and middle finger causing
appearance of
v. pointing index sign
+ Loss of motor and sensory, same like low lesion

22. Axillary nerve injury


A. Can be caused by fracture of neck of femur - f
B. pts able to abduct his arm at 30 degrees – f -
C. Associated with small patch of numbness over deltoid area - t
D. Unable to abduct the shoulder - t
E. Can only be treated through surgical intervention - f

Anatomy: it supply deltoid  abduction of the arm (deep branch)


Teres minor external rotation (superficial branch)
Sensory: lat upper arm at the regimental badge area

Cause: shoulder dislocation, frac of neck of humerus


Effect: unable to abduct the arm from 15-90 degree.
vi. Flat shoulder
vii. loss of sensation at regimental batch (5 finger breath from the tip of acromion)
Tx: recover spontaneously
But if after 8wks still not recover >> explore and nerve grafting
If fail >> tendon transfer

23. Radial nerve injury


A. Low lesions are usually due to fracture or dislocations at the elbow - t
B. Patient can extend the MCP joint in low lesion - f
C. very high lesions occur due to fracture of humerus - f
D. In high lesion, wrist drop is common manifestation - t
E. Small patch of sensory loss on the back of the hand at the base of the thumb can be
seen in low lesion - f

Anatomy : radial.n supplies the whole extensor muscle of the upper limb. From top it runs
through the triangular interval with deep artery of the arm and passing through the spiral
groove (radial grove) of humerus. Then it divide at the elbow into 1. PIN (post
interossious.n)-purely motor. This nerve runs through the supinator.m. 2. Sup. Radial.n-
sensory, to the dorsal lat. 3 and half finger.

Arm
Motor: triceps  extend the elbow
Anconeous
Sensory: lat arm (via inferior lat cutaneous.n)
Post arm (via post cutaneous.n)
Forearm
Motor – mobile WAD
Brachioradialis
Supply by radial.n- mainly for
Extensor carpi radialis longus
Extensor carpi radialis brevis wrist extension
Superficial extensor
i. Extensor carpi ulnaris
ii. Extensor digiti minimi
iii. Extensor digitorum

Deep extensor Supply by PIN- mainly for finger


i. Supinator extension
ii. Abductor pollicis longus
iii. Extensor pollicis longus
iv. Extensor pollicis brevis
v. Extensor indicis
Sensory: post forearm
Hands
Motor: NONE
Sensory: dorsal lat 3 and half finger (via superficial radial.n) – specific point at the 1st finger
web

Low lesion:
Cause: frac or dislocation at elbow, open wound frac of the arm
Effect: unable to extend MCPJ finger drop bcoz only the PIN is affected
i. Sensory is still intact

High lesion:
Cause: frac of humerus or pressure (Saturday night palsy)
Effect: unable to extend the wrist wrist drop bcoz the radial.n is effected. So it involve the
i. mobile WAD  wrist extension, superficial and deep extensor  finger
extension
ii. Sensory loss (dorsal aspect of tumb, index, middle finger and half of ring finger)

Very high lesion:


Cause: pressure (crutch palsy)
Effect: unable to extend the elbow bcoz also involve the tricep
i. tricep muscle wasting
ii. loss of motor and sensory just like high lesion

24. Ulnar nerve injury


A. Low lesion may be caused by pressure or a laceration at the wrist - t
B. Claw hand present in the low lesion - t
C. High lesions occur with elbow fractures - t
D. Sensation is loss over the ulnar medial one and a half fingers in low lesion - t
E. Patient is difficult to make full grip and pinch - t

Anatomy - ulnar nerve supply the hypothenar muscle (consist of: abductor digiti minimi,
flexor digiti minimi, opponens digiti minimi digiti minimi stand for little finger and it fn
base on it name), intrinsic muscle (consist of: dorsal interossie finger abduction, palmar
interossei finger abduction, lumbricals fn flex MCP and extend PIP), adductor pollicis
adduction of the tumb, flexor pollicis brevis (together with median.n)
Ulnar.n sensory supply: little and half of ring finger on palmar and dorsal (specific point-at
tip of little finger)
Low lesion
- Cause: pressure or laceration at the wrist
- Effect: ulnar true clawed hand (hyperextend of MCPJ and flexion of IPJ of little and
ring finger)- this is how to differentiate btw low lesion or high lesion
i. Hypothenar muscle wasting
ii. Weak finger abduction
iii. Loss of thumb adduction difficulty to pinch
iv. Loss of sensation of little and half of ring finger, on palmar and dorsal side.

High lesion
- Cause: elbow frac, pressure after lying with the flex elbow and pressing on the bed,
cubitus valgus due to malunion
- Effect: less claw hand (ulnar paradox)- bcoz medial aspect of flexor digitorum
profundus is
- supply by ulnar.n, it fn is to flex little and ring finger.
- In low lesion (lesion at the wrist) flexor digitorum profundus is not affected as it’s
supply by ulnar.n at higher up there is flexion of the little and ring finger causing
appearance of claw hand. But there will be minimal claw hand in higher lesion
because the flexor digitorum profundus also affected (from drfairudz)
- + Loss of motor and sensory, same like in low lesion

25 Regarding the clubfoot.


A Can occur in the association with arthrogryposis
T  Apley’s pg 241.
 Similar deformities are seen with myelomeningocele and arthrogryposis.(
http://emedicine.medscape.com/article/1237077-overview)
B Female is more predominant
F Boy : girl = 2:1 (apley’s pg 241)
C 50% bilateral involvement
T  The male-to-female ratio is 2:1. Bilateral involvement is found in 30-50% of cases.
There is a 10% chance of a subsequent child being affected if the parents already have
a child with a clubfoot. (http://emedicine.medscape.com/article/1237077-overview).
 The condition is bilateral in one third of cases. (apley’s pg 241).
D Deformity described as fixed equines,heel varus,fore foot and mid foot adduction
T Apley’s pg 241 (first line).
E Surgical correction is gold standard
F Surgical correction in resistant cases. (Apley’s pg 241). Earliest treatment is by conservatively.
Repeat manipulation and adhesive strapping or POP. Surgical releases of Achilles tendon
needed to complete correction.

26 Imaging of osteomyelitis
A Earliest changes include minimal periosteal destruction and thickening
F First 10 days, no abnormalities , 2nd week rarefaction of metaphysic and periosteal new bone
formation. (Apley’s pg 19).
B Usefulness of radionuclide scanning limited by an overall lack of specificity and marginal
sensitivity
F http://emedicine.medscape.com/article/785020-diagnosis nie xsure xdapat cari explaination.
C MRI can also evaluate extents of infectious problems
F  MRI help to distinguish between bone and soft tissue infection. (apley’s pg 19).
D CT scan distinguish between soft tissue and bone infections and aids in biopsy and aspiration
site
F  MRI is use to distinguish (Apley’s pg 19).
 Aspiration guided by ultrasound (http://emedicine.medscape.com/article/785020-
diagnosis).
 CT to look for abnormal calcification, ossification and intracorticol abnormalities.
Often choose when MRI unavailable (http://emedicine.medscape.com/article/785020-
diagnosis).
E Sclerotic changes and periosteal new bone formation suggest acute
F Nie xsure. Sclerotic and cortical thickening seen in chronic (Apley’s pg 20), periosteal new bone
formation seen end of second week (acute stage la kan???) (Apley’s pg 19).

2 Achilles tendon rupture


7
A Common in people > 40 years old
T Apley’s pg 250
B Simmond’s test positive
T  http://web.jbjs.org.uk/cgi/reprint/74-B/2/314.pdf
 http://www.youtube.com/watch?v=AmDi08rlR3I
 Used to test for rupture of tendo achillis. Positive when no movement of plantarflexion
when the culf is squeeze or pushed while the patient is prone and the foot is hanging by
the side of the bed.
C Result in difficult to walk tip-toe
T Patient unable to tiptoe. (Apley’s pg 250).
D Cannot be treat non-operatively
F  Immobilization by plaster is still needed with or without surgery and is worn for 8
weeks with the foot in equines. Shoe with raised heel worn for a further 6 weeks.
(apley’s pg 250).
 Lockable brace is a more sophisticated alternative. (apley’s pg 250).
E Treatment via surgical reconstruction will result in wound dehiscence as
T  http://www.emedicinehealth.com/achilles_tendon_rupture/page7_em.htm#Surgery
 http://emedicine.medscape.com/article/85024-treatment

28 Adhesive capsulitis (frozen shoulder)


A Commonly occurs in patient aged 40-60 years old
T Apley’s pg 147.
B Is presented with loss of shoulder movement in all directions
T  Apley’s pd 147.
 http://www.ncbi.nlm.nih.gov/pubmedhealth/PMH0001490/
 http://www.medicinenet.com/frozen_shoulder/article.htm
C Is caused by a trauma to the shoulder
T Apley’s pg 147. As part of history of the patient.
D Is a self limiting disease
T Resolves spontaneously after 18 months. (apley’s pg 147)
E In association with diabetes mellitus would result in poor recovery
T Xjumpa sumber2 yg reliable…..
 http://www.ncbi.nlm.nih.gov/pubmedhealth/PMH0001490/ ……diabetes is one of the
risk factor, so should treated/controlled first to get proper/faster recovery.
29. Pseudogout
A. Is presented as an acute attack of arthritis which is self limiting T
B. Is due to deposition of monosodium urate crystal F
C. Is demonstrated by positive birefringent in polarized light microscopy T
D. Can result in the formation of tophi F
E. Is shown as chondrocalcinosis on plain x-ray T

A. Patient typicically middle aged women, complaint acute pain and swelling in 1 large
joint,usually knee. Untreated condition last for few weeks and then subsides spontaneously.
From the reasoning above, the answer for (a) is TRUE
(Ref: Apley, 3rd edition 2008, page 39)

B. Calcium pyrophosphate dehydrate deposition occurred at :


- Chondrocalcinosis
- Pseudogout
- Chronic pyrophosphate arthropathy
- Monosodium urate crystal = gout
- From the reasoning above, the answer for (b) is FALSE
(Ref: Apley, 3rd edition 2008, page 39 and 37)

Diagnosis of pseudogout can be confirmed by finding positive birefringent crystals in


synovial fluid.
(Ref: Apley, 3rd edition 2008, page 40)

C. Polarized light microscopy of synovial fluid shows negative birefringent crystal in gout
(Ref: Oxford handbook medicine, 7th edition, page 534)
From the reasoning above, the answer for (c) is TRUE

D. During chronic gout, tophi appear around joint, olecranon and pinna ear. It can ulcerate and
discharge is chalky material
a. Pseudogout only has pain and swelling of joint.
b. (Ref: Apley, 3rd edition 2008, page 39)
c. From the reasoning above, the answer for (d) is FALSE

E. Xray may show signs of chondrocalcinosis


d. (Ref: Apley, 3rd edition 2008, page 40)
e. From the reasoning above, the answer for (e) is TRUE

30. Trendelenburg’s sign in hip examination results is positive when conducted on patient with:
A. Non-union femoral neck fracture F
B. Poliomyelitis affecting hip abductor muscles T
C. Chronic hip joint dislocation T
D. Fracture of lesser trochanter of the femur F
E. Ankylosed hip F

Trendelenburg test used to assess stability. Ask patient to stand by 1 leg, unassisted, lift the
other leg by bending the knee.
Normal: pelvis rise at the lift leg (hip is stable by abductors muscle)
Abnormal: Pelvis drop at the lift leg.
Positive:
a. dislocation of hip
b. Weakness abductors muscle
c. Shortening femoral neck
d. painful disorder of hip
(Ref: Apley, 3rd edition 2008, page 202)

Answer for question A and D was not stated anywhere. I have asked this question to Dr
Ramli Baba, he said that, Trendelenburg test cannot be done on fracture patient as they
already pain, how they could stand on the affected limb.

31. Fractures:
A. When due to repetitive stress are called pathological fractures F
B. When due to forcible traction by a tendon are referred to as avulsion fractures. T
C. Are classified to as comminuted when there is more than 1 fragment. F
D. Will usually unite even the bone ends lie side by side with fractures surfaces making
no contact at all. F
E. In adults when incomplete is referred to as greenstick fractures. F

Repetitive stress is called stress fractures


Pathological fracture is normal stress acting on abnormal weakened bone
(Ref: Apley, 3rd edition 2008, page 266)

An avulsion fracture is an injury to the bone in a place where a tendon or ligament attaches
to the bone. When an avulsion fracture occurs, the tendon or ligament pulls off a piece of
the bone.
(Ref: http://orthopedics.about.com/od/brokenbones/a/avulsion.htm)

Comminuted fracture is more than 2 fragment, poor interlocking of fragment caused it to be


unstable.
(Ref: Apley, 3rd edition 2008, page 267)

Fracture can never unite if the end surface does not have any contact at all.
Cause of non union includes distraction and separation fragment (end surface no contact)
(Ref: Apley, 3rd edition 2008, page 270)

Greenstick fractures, the bone are bent. Seen in children, whose bone are more
springy/pliable compared to adult.
Adult usually complete fracture
(Ref: Apley, 3rd edition 2008, page 267)

32. Factors associated with non union of fractures include:


A. Fracture angulation T
B. Smoking T
C. Splintage with Plaster of Paris F
D. Interposition of periosteum between the fragments.F
E. Osteoporosis T
32. Fracture caused non union
i. Distraction and separation fragment
ii. Interposition soft tissue between fragment
iii. Excessive movement
iv. Poor blood supply
v. Severe damage soft tissue that makes them non viable
vi. Infection-DM
vii. Abnormal bone (osteoporosis)
viii. Smoking, alcohol (Ref: http://en.wikipedia.org/wiki/Nonunion)
ix. Obese
(Ref: Apley, 3rd edition 2008, page 270)
http://apps.djoglobal.com/bonestim/patients/fracture.asp)

33.

Monteggia fracture - originally it is a fracture of the shaft of the ulnar (A/T, B/F) associated with
disruption of the proximal radioulnar joint (D/F) and dislocation of radiocapitellar joint but nowadays it
may includes olecranon fracture combined with radial head dislocation (C/T).

Aims of treatment is to restore the length of fractured ulnar, only then dislocated joint be fully reduced
and remain stable by means of operation with plates and screws in adults (E/T). (The unstable # in
Monteggia is actually means by the joint dislocation only can be reduced after the ulnar # has been fixed,
so ORIF is a definitive treatment!!!)

Source – Apley’s Concise System of Orthopaedics & Fractures,


3rd Edition, Solomon: Warwick: Nayagam (pg 322-323)
34. Compartment Syndrome:
A. Occur when bleeding or edema increases the pressure in one of the osteofascial
compartment beyond systemic blood pressure. F
B. Is characterized by excruciating pain F
C. Is a complication of a tight plaster cast T
D. Is adequately treated by analgesics and elevating the limb. F
E. Cause little long term disability in untreated condition. F

Compartment syndrome is actually due to increase osteofacial compartment pressure that disrupts
the capillary blood flow to the compartment and lead to ischemia (A/F ) and not really associate with
systemic blood pressure. This process will undergo vicious cycle until 12h where the nerve and the
muscle become necrosis. The nerve is capable of regeneration but muscle, once infracted, can never
recover and will be replaced by inelastic fibrous tissue leads to Volkmann’s ischemic contracture.
Compartment syndrome also can occur in a swelling limb which has been put inside thought plaster
cast (C/T).
Source – Apley’s Concise System of Orthopaedics & Fractures,
3rd Edition, Solomon: Warwick: Nayagam (pg 322-323)

The classical features of ischemia are pain, parasthesia, pallor, paralysis and pulselessness and
usually they describe the pain as a ‘bursting’ sensation (B/F).

Source – Apley’s Concise System of Orthopaedics & Fractures,


3rd Edition, Solomon: Warwick: Nayagam (pg 322-323)
Aim of treatment in compartment syndrome is to relieve the high pressure by means of prompt
decompression. Casts, bandages and dressings must be completely removed and the limb should be in
flat position because by elevating the limbs will cause a further decrease in end-capillary pressure
and aggravates the ischemia(D/T). Open fasciotomy is performed to open the compartment so that
the pressure will be reduce. The wound should be left open and inspected 2 days later, if there is
muscle necrosis, debridement should be done but if the tissue is healthy, the wound can be sutured,
skin-grafted or allowed to heal by secondary intention.

Source – Apley’s Concise System of Orthopaedics & Fractures,


3rd Edition, Solomon: Warwick: Nayagam (pg 322-323)

35. Regarding traumatic paraplegia.


A. Complete paralysis and anaesthesia is a consequence of spinal shock. T
B. Bladder training is required T
C. Flexion contracture cannot be avoided. F
D. Bedsore developed after 2 day. F
E. Emotional rehabilitation is not vital F

Complete transaction of the cord results in either paraplegia/quadrapelgia. Initially there will be
complete paralysis and anaesthesia with loss of the anal reflex result from spinal shock (A/T). After
24h, the anal reflex returns and the neurological deficit still persist, we can assume it is complete cord
lesion/injury. Gradually, UMN lesion features will appear: spastic paralysis & hypereflexia.

Incomplete transaction – partial motor and sensory loss below the level of lesion with signs varies
according to the part of cord has been damaged.

Cauda equina injury – features of LMN lesion: flaccid paralysis.

Source – Apley’s Concise System of Orthopaedics & Fractures,


3rd Edition, Solomon: Warwick: Nayagam (pg 351)

Due to loss of nerve innervations to the bladder, it can cause bladder distension, overflow urinary
incontinence and infections. To prevent those complications, bladder trained should be initiated as
soon as possible for example: intermittent catheterization under sterile/clean condition and
continuous closed drainage with disposable bag changed twice a week. In cauda equina injury, local
reflex will be lost and there will be no bladder emptying, so the patient needs to empty their own
bladder by manual suprapubic pressure method (B/T).

Source – Apley’s Concise System of Orthopaedics & Fractures,


3rd Edition, Solomon: Warwick: Nayagam (pg 351-352)
In prolonged immobilization, untreated muscle
paralysis will cause severe flexion contracture
and it is actually preventable by moving the If flexion contracture still or are allow to
joints (C/F) passively through their full range develop; tenotomies, neurectomies,
twice daily. rhizotomies or intrathecal injection of
alcohol are the possible solution.

If the lesion is below the cervical cord,


standing and walking within 3 months is i.
important to prevent contracture. ii. Source – Apley’s Concise System of Orthopaedics
Calipers usually needed to keep the & Fractures,
knees straight and the feet plantigrade. 1. 3rd Edition, Solomon: Warwick:
The upper limbs must be trained until they Nayagam (pg 351)
develop sufficient power to enable the
patient to use crutches and a wheelchair.

Bedsores or pressure sores may develop just within a few hours of immobilize patient (D/F) especially
who had spinal injury due to anaesthetic skin. Initially, meticulous nursing of the skin is important to
prevent pressure sores usually by gentle rolling onto the patient side and the back is carefully washed,
dried and powdered for every 2 hours. After a few weeks, the patient may turn in bed by their ownself
to relieve skin pressure intermittently guided by proper education from the healthcare staffs.

Source – Apley’s Concise System of Orthopaedics & Fractures,


3rd Edition, Solomon: Warwick: Nayagam (pg 351)

The morale of paraplegic patient is a liable to reach low ebb or depression and to restore the patient
self-confidence is an important part of treatment. The earlier the patients get up the better the
prognosis, and they must be trained for a new job as quickly as possible to improve their quality of
life. (E/F)
Source – Apley’s Concise System of Orthopaedics & Fractures,
3rd Edition, Solomon: Warwick: Nayagam (pg 352)

36. Neck of femur fracture


A. Avascular necrosis is late complication F
B. Occur commonly in older men T
C. Occur commonly in osteoporotic T
D. Surgery is not indicated if displaced fracture F
E. Garden type II is complete with minimal displacement. F
Garden’s Classification of femoral neck fractures

 Grade I is an incomplete or valgus


impacted fracture.
 Grade II is a complete fracture
without bone displacement. (E/F)
 Grade III is a complete fracture
with partial/minimal/moderate
displacement of the fracture
fragments.
 Grade IV is a complete fracture
with total displacement of the
fracture fragments.

Source – Apley’s Concise System of Orthopaedics & Fractures,

2. 3rd Edition, Solomon: Warwick:


Nayagam (pg 362-
363)http://emedicine.medscape.com/ar
ticle/86659-overview

Operative treatment is almost mandatory because in displaced fracture, union will not be occur
without internal fixation (D/F) and it is important for elderly to mobilize and be active without delay to
prevent pulmonary complications and pressure sores. Eventhough incomplete impacted fracture can be
left to unite, internal fixation is always useful as there is always a risk to become displaced even while
lying on bed.
Source – Apley’s Concise System of Orthopaedics & Fractures,
3rd Edition, Solomon: Warwick: Nayagam (pg 363)
Avascular necrosis is really an early complication of bone injury because ischemia occurs during the
first few hours following fracture or dislocation. Especially at the head of femur following femoral neck
fracture (A/F) and hip dislocation, proximal part of scaphoid, the lunate following dislocation and body
of talus after its neck fracture. (in chapter 6 apleys it is actually classified under late complication!
Hmmm…..)
In femoral head necrosis, 30% of patients will develop following displaced fractures and 10% following
undisplaced fractures. It is because when there is fracture at the neck of the femur, the branches from
the nutrient artery are severed, the retinacular vessels from the capsule are torn and the remaining
blood supply from ligamentum teres may be insufficient to prevent ischemia of the femoral head. All of
those disruptions will lead to bone dies and eventually collapse.

Source – Apley’s Concise System of Orthopaedics & Fractures,


3rd Edition, Solomon: Warwick: Nayagam (pg 299 & 365)

37
A F Medial meniscus injury is more common than lateral meniscus injury with the ratio of 3:1
(Netter’s concise orthopedic anatomy, 2nd edition, page 328)
B T There usually bleeding & swelling into the tissue surrounding the knee in collateral
ligament tear. The tear may also caused bleeding into the joint itself.
http://www.orthogate.org/patient-education/knee/collateral-ligament-injuries.html
C F The plan for surgical or non-surgical of ACL tear depends on age, skeletal maturity,
xtvt/skill level, a/w meniscal & ligamentous injury, frequency of instability, pts
compliance & motivation. If there is an isolated tear of ACL, treat it by early op
reconstruction if the individual is a professional sportsman. But in all other cases, it is
more prudent to follow the conservative management.
But if there is combined ACL & collateral ligament injury, start the treatment with joint
bracing & physiotherapy to restore good ROM then followed by ACL reonstruction.
Usually the surgical management is delayed 4-6 weeks after the injury because early
surgery will cause arthrofibrosis
(Apley, 3rd edition, page 375&376; seminar sport injury dr faisal)
D F Swelling in cruciate ligament tear appears almost immediately compare to swelling in
meniscus injury. The swelling in meniscus injury appears some hours later or perhaps the
following day. (Apley, 3rd edition, page 227&374)
E T Posterior tibial sagging sign can be detect laterally by putting the patient in supine with
the knee and hip in 90˚ flexion. The gravity will pulls the tibia posteriorly. In the case of
PCL tear, the tibial falls even or behind the femoral condyle. Compare with the opposite
knee.
http://emedicine.medscape.com/article/90514-overview

38
A F Osteosarcoma has bimodal age distribution. 1st peak is during adolescent, coinciding with
the pubertal growth spurt. 2nd peak is in adults >65 years of age and it is more likely
represent a secondary malignancy (Paget’s disease) (Pediatric and adolescent
osteosarcoma by Norman Jaffe, Oyvind S.Bruland, page 3)
Osteosarcoma most commonly found in children and adolescent (Apley 3rd edition, page
91) (10-20 years)
B F Most commonly it affects the long bones metaphysic especially around the knee, proximal
end of humerus (Apley 3rd edition, page 91)
C F The incidence of osteosarcoma higher in boys than in girls (Pediatric and adolescent
osteosarcoma by Norman Jaffe, Oyvind S.Bruland, page 3).
D T Osteosarcoma is a primary bone cancer, means the cancer originates in the bone itself
(http://www.boneandcancerfoundation.org/pdfs/Osteosarcoma-2.pdf)
E F Spreading to regional lymph nodes is almost never occur. This probably due to the poor
lymphatic supply to the bone and most important the tumor is so vascular therefore
hematogenous spread is more common.
Hematogenous spread Pulmonary metastasis (Most common & occur early (10%))
Direct spread to the surrounding soft tissues & along the medullary cavity of a long
bone
http://www.medic.usm.my/~pathology/bonepath/bonepath/Osteosarcoma.html

39
A F Neurapraxia is a REVERSIBLE block to the nerve conduction which there is loss of
sensory/motor power.
After few days or weeks, there will be spontaneous recovery of neurapraxia.
The nerve is intact but mechanical pressure caused demyelination of axons in a limited
segments (Apley, 3rd edition, page 110)
B T Neurotmesis is complete disruption of the nerve, such as may occur in an open wound.
There are disruption of epineurium, perineurium, endoneurium, myelin sheath and axon.
(Involved all nerve layers) (Netter’s concise orthopaedic anatomy, 2nd edition, page 22)
It will never recover without surgical intervention, poor prognosis (Apley, 3rd edition,
page111)
C F In axonetmesis, there is loss of conduction but the nerve is still in continuity and the
neural tubes intact (epineurium layer still intact). Axonal regeneration will occur within
hours of nerve damage. (Apley, 3rd edition, page 111 & Netter’s concise orthopedic
anatomy, 2nd edition, page22)
D T Axonal regeneration starts within hours of nerve damage. The new axonal processes
grow at a speed of 1-2mm per day (Apley, 3rd edition, page 111)

E T Axonotmesis usually seen after closed fracture and dislocations (Apley, 3rd edition, page
110)

40
A T Gout is more widespread in men than in women (ratio 20:1),usually men at the age of
>30 years, rarely seen in female before the menopause (Apley 3rd edition, chapter 4, page
37&38)
B F Myeloproliferative disease is classified into secondary gout which comprises only 5%
causes of gout (others 2ndary gout due to administration of diuretics or renal failure). The
95% is due to primary gout where there is absence of any obvious cause and may be due
to constitutional under-excretion or over-production of urate (Apley 3rd edition, chapter
4, page 38)
C F GOUT- Examination of aspirated joint fluid under polarizing microscope shows needle-
shape, negative birefringent monosodium urate crystals
PSEUDOGOUT – examination of aspirated joint fluid under polarizing microscope shows
rhomboid shape (rectangular), positive birefringent calcium pyrophosphate crystals
(Family medicine By David R. Rudy, page 162)
Birefringent definition= the quality of transmitting light unequally in different directions
(Dorland’s medical dictionary).
Birefringent (double refraction, pembiasan berganda) = there is decomposition of a ray
of light into two ray after pass through the anistropic materials (e.g: calcium
pyrophosphate crystal)( http://en.wikipedia.org/wiki/Birefringence)
D T Gout can be confused with septic arthritis in an acute attack because they have similar
presentation, acute onset of hot severe joint pain, extremely tender, fever, chills, and
malaise. (Apley 3rd edition, page 22&38)
Take careful history taking to identify the risk factors of septic arthritis (exposure to
gonorrhea, recent puncture wound over the joint, systemic signs of disseminated
infection) and gout (hyperlipidemia, hypertension, hyperTG, kidney failure, obese, insulin
resistance, alcohol intake). (http://emedicine.medscape.com/article/808628-overview)
Gout also occasionally can co-exist with septic arthritis. The details differences of joint
fluid characteristics between gout and septic arthritis, refer CPG, management of gout,
October 2008, MOH; page 17
E F Acute gout: NSAIDs- rapidly effective in relieving pain & reducing inflammation. E.g:
diclofenac, indomethacin & ketoprofen. Avoid aspirin (causes urate retention unless
given in very high doses). Caution in pts that having hx of peptic ulcer disease, HPT, renal
impairment & cardiac failure
Other 1st line agents: steroid and low-dose colchicines.
Colchicine is an alternative drug for those whom NSAID & COX-2 inhibitors are
contraindicated. Colchicines acts as an anti-inflammatory drugs. Use with low doses
because of its side effects such as nausea, vomiting, abdominal pain & profuse diarrhea
Allopurinol used as a prophylaxis of gout when hyperuricemia. Because the allopurinol is
xanthine oxidase inhibitor prevent production of uric acid.
Allopurinol cannot be used in acute attack because it may precipitate or worsen an acute
attack of gout. It should be initiated only with concurrent use of colchicines or NSAIDs.
(Apley, 3rd edition, page 39; Family practice examination and board review by Mark
Graber, Jason K.Wilbur, page382)
Other uricosuric agents: probenecid or sulphinpyrazone can be used if renal function is
normal
CPG, Management of gout October 2008

41. Rheumatoid arthritis


A. ESR increase T
Investigation for RA :
i. in active phase the erythrocyte sedimentation rate (ESR) is raised and C- reactive protein
is present.
ii. serological test for rheumatoid factor are positive in 80% of ptnt; sometimes antinuclear
factors also are present.
( appley’s , p-28, para- investigations )

B. Positive rheumatoid factors makes diagnosis certain F

Minimal criteria for diagnosing RA :


i. Bilateral , symmetrical poluarthritis
ii. Involving the proximal joint of the hands or feet
iii. Present for at least 6 weeks.
- In addition, there are subcutaneous nodules or periarticular erosion on x ray, the
diagnosis is certain.
A positive test for rheumatoid factor in the absence of the above features is not sufficient to
diagnose rheumatoid arthritis, nor does a negative test exclude the diagnosis if all the other
features are present.
(appley’s , p-28 , para- diagnosis )
C. Periarticular erosions on xrays are characteristic

Explanation as above

D. Diseases show relentless progression in majority of cases T


i. In 80% follows periodic course, with intermittent “flares “ during which sx and signs
are more severe. With time these attacks occur less frequently and the disease may
become almost quiescent; by then, joint are often permanently damaged.
ii. In 5% of cases there is relentless progression of the disease, with increasing
inflammatory activity, joint destruction, muscle wasting and visceral involvement.
iii. In 10%( usually men over 55yr sx starts explosively but, rather paradoxically, the
condition tends to subside and follows a relatively mild course)
(appley’s , p-29, para – course)

E. Splintage of inflamed joint is contraindicated as it causes stiffness. F


i. physiotherapy is still important- one of the oldest methods of treating inflammation.
ii. during acute flare up, the pntn may benefit from a few weeks’s rest; gentle active
and passive exercise are kept up and care should be taken to prevent postural
deformities.
iii. Sometimes a week or two of continuous splintage ( for the wrist or knees) is all that
needed; night splints can be used intermittently at any stage of the disease.
( appley’s , p- 30 , para – 5)

42. Traumatic posterior dislocation of the hip joint


A. Occur following a dashboard injury T
i. post dislocation usually occurs in road traffic accident when someone seated in a
truck or car is thrown forwarrrds, striking the knee against the dashboard.
ii. The femur is thrust upwards and the femoral head is forced out of its socket; often a
piece of bone at the back of the acetabulum is sheared off ( frrracture dislocation)
(appley’s , p- 361, para – posterior dislocation )

B. Cause the hip to be positioned into externally rotated and flexed F


i. in a straightforward case the diagnosis is easy: the leg is short and lies adducted,
internally rotated and slightly flexed
(appley’s , p-361, para – special features )

C. Will result in avascular necrosis of the femoral head as its late complication T

Complication of post dislocation of the hip :


i. Sciatic nerve injury
- damaged in 10-20% of injury
- usually recovers, recovery often takes months and in meantime limb must be
protected form injury and ankle splinted to overcome foot -drop
ii. avascular necrosis
- blood supply of femoral head is seriuously impaired
- if there is a small necrotic segment, realignment osteotomy is method of choice.
- Younger ptnt, choice is between femoral head replacement or hip arthrodesis
- Ptnt >50 yrs – thp is better.
iii. Osteoarthritis
Secondary OA is not uncommon and is due to:
- cartilage damage at the time of the dislocation
- the presence of retained fragments in the joint (3)ischemic necrosis of the
femoral head.
(appley’s , p- 361, para – complication )

D. Will be associated with femoral nerve injury F


explanation as above

E. Require an urgent open reduction F


- the dislocation must be reduced under general anesthesia.
- an assistant steadies the pelvis; the surgeon flexes the ptnt’s hip and knee to 90◦ and
pulls the thigh vertically upwards.
- x rays essential to confirm reduction and to exclude fractures.
- if it is suspected that bone fragment have been trapped in the joint, CT is needed.
(appley’s , p- 361 , para – treatment )

43. In acute hematogenous osteomyelitis:


A. Adults are more commonly affected than children F
i. acute OM almost invariably occurs in children; when adults are affected it may be
because of compromised host resistance due to debilitation, disease or drugs ( e.g.
immunosuppressive therapy)
(appley’s , p- 17 )

B. Can result in septic arthritis if occur in young children T


Complication of OM :
i. spread – infx may spread to joint ( septic arthritis) or to other bones ( mets OM)
ii. growth disturbance – if the physis is damaged, there may later be shortening or
deformity.
iii. Persistent infx – may result in chronic OM
(appley’s p-19, para – complications)

C. Plain x-rays changes are evident within 1 week T


i. for the first 10 days, x rays shows no abnormality. However, radioisotope scans may
show increased activity( non specific sign of acute inflammation)
ii. by the end of the second week there may be early radiographic signd of rarefaction
of the metaphysic and periosteal new bone formation.
iii. later still, if treatment is delayed bone may appear increasingly ragged.
iv. with healing there is sclerosis and thickening of the cortex.
(appley’s, p- 19, para – imaging )

D. C-reactive protein will be positive T


Laboratory Studies - The following studies are indicated in patients with osteomyelitis:
CBC count: The WBC count may be elevated, but it is frequently normal.
- A leftward shift is common with increased polymorphonuclear leukocyte counts.
- The C-reactive protein level is usually elevated and nonspecific; this study may be
more useful than the erythrocyte sedimentation rate (ESR) because it reveals
elevation earlier.
- The ESR is usually elevated (90%); however, this finding is clinically nonspecific.
- CRP and ESR have limited roles in the setting of chronic osteomyelitis and are often
normal
Culture: Superficial wound or sinus tract cultures often do not correlate with the
bacteria that is causing osteomyelitis and have limited use. Blood culture results are
positive in approximately 50% of patients with hematogenous osteomyelitis. However, a
positive blood culture may preclude the need for further invasive procedures to isolate
the organism. Bone cultures from biopsy or aspiration have a diagnostic yield of
approximately 77% across all studies.
(http://emedicine.medscape.com/article/785020-treatment)

E. Emphirical antibiotic is given without knowing the causative organism T


- Treatment for osteomyelitis involves the following:
o Initiation of intravenous antibiotics that penetrate bone and joint cavities
o Referral of the patient to an orthopedist or general surgeon
o Possible medical infectious disease consultation
- Select the appropriate antibiotics using direct culture results in samples from the
infected site, whenever possible.
- Empiric therapy is often initiated on the basis of the patient's age and the clinical
presentation.
- Empiric therapy should always include coverage for S aureus and consideration of
CA-MRSA.
- Further surgical management may involve removal of the nidus of infection,
implantation of antibiotic beads or pumps, hyperbaric oxygen therapy,11,12 or other
modalities.
(http://emedicine.medscape.com/article/785020-treatment)

44. The axillary nerve injury

- The axillary nerve supplies three muscles; deltoid (a muscle of the shoulder), teres
minor (one of the rotator cuff muscles) and the long head of the triceps brachii (an
elbow extensor).[1]
- The axillary nerve also carries sensory information from the shoulder joint, as well as the
skin covering the inferior region of the deltoid muscle - the "regimental badge" area
(which is innervated by the Superior Lateral Cutaneous Nerve branch of the Axillary
nerve).
- When the axillary nerve splits off from the posterior cord, the continuation of the cord is
the radial nerve.
(http://en.wikipedia.org/wiki/Axillary_nerve)

A. associated with fracture of the humeral head T??


The axillary nerve may be injured in anterior-inferior dislocations of the shoulder joint,
compression of the axilla with a crutch or fracture of the surgical neck of the humerus
(http://en.wikipedia.org/wiki/Axillary_nerve)

B. Is a complication following traumatic shoulder dislocation T


- traumatic shoulder dislocation is common; humeral head displacement is usually
anterior, less often posterior
- compliacation of dislocation of the shoulder is nerve injury whereby the axillary nerve
maybe injured.
( appley’s, p- 306, para – dislocation of shoulder and nerve injury )
C. Results in weakness in shoulder abduction T
- The axillary nerve may be injured in anterior-inferior dislocations of the shoulder
joint, compression of the axilla with a crutch or fracture of the surgical neck of the
humerus. Injury to the nerve results in:
- Paralysis of the teres minor muscle and deltoid muscle , resulting in loss of abduction
of arm (from 15-90 degrees), weak flexion, extension, and rotation of shoulder.
Paralysis of deltoid & teres minor results in Flat shoulder deformity.
- Loss of sensation in the skin over a small part of the lateral upper arm.
(http://en.wikipedia.org/wiki/Axillary_nerve)

D. Results in numbness over the proximal forearm F


- the patient is unable to contract the deltoid muscle and there maybe a small patch of
anesthesia over the muscle( regimental batch)
(appley’s , p- 306 , para – nerve injury )

E. Occur following a closed injury is often neupraxia T


- the lesion is usually a neurapraxia , which recovers spontaneously after a few weeks
( appley’s, p- 306, para- nerve injury )

45. Prognostic features for Perthes disease in a child is based on:

A. Age T
= following the age, if the onset of the Perthes’ disease under the age of 6, it is favorable
prognostic sign where they need no active treatment and have to be put under follow up.
While, if the onset >6 years old, it is unfavorable sign (poor prognosis) and they need
treatment by containment of the femoral head. (‘containment’= keeping the femoral head
well seated within the acetabulum)
(ref: Apley’s Concise System of Orthopaedics and Fractures)
B. Sex T
= Perthes’ disease affects boys four times more frequently than girls. (M: F=4:1).
(ref: Apley’s Concise System of Orthopaedics and Fractures)
C. Degree of the head involvement T
= involvement of femoral head also one of the prognostic feature in Perthes disease which
we can follow Herring classification. Herring classification is recommended as one of the
prognostic grading system, based on the severity of structural disintegration of the lateral
pillar of the femoral epiphysis apart from Salter-Thomson and Catterall staging. It compares
the height of lateral epiphyseal pillar to the height of the contra-lateral epiphysis. (Group A:
there is no collapse of the lateral pillar and there is little density changes; Group B: lateral
pillar margins has >50% of original height; Group C: collapse of lateral pillar >50 %.). If only
partial involvement of the femoral head, it give good prognosis.
Catterall classification is based on radiographic appearances and specifies 4 groups during
the period of greatest bone loss. Catterall staging is as follows:
Stage I — Histologic and clinical diagnosis without radiographic findings
Stage II — Sclerosis with or without cystic changes with preservation of the contour
and surface of femoral head
Stage III — Loss of structural integrity of the femoral head
Stage IV — Loss of structural integrity of the acetabulum in addition
The Salter-Thomson classification simplifies the Catterall classifications by reducing the
groups to 2. The first, called group A, includes Catterall groups I and II; for patients in this
group, less than 50% of the head is involved. The second, called group B, includes Catterall
groups III and IV; for patients in this group, more than 50% of the head is involved. For both
classifications, if less than 50% of the ball is involved, the prognosis is better, whereas if
more than 50% is involved, the prognosis is potentially poor.
(ref: Apley’s Concise System of Orthopaedics and Fractures,
http://www.wheelessonline.com/ortho/radiographic_evaluation_of_perthes_disease,
http://emedicine.medscape.com/article/410482-overview.)

D. Duration of the hip pain F


= Duration of the hip pain did not become one of the prognostic features of Perthes’ disease
as usually the children will feel intermittent pain for several weeks to months as they often
did not complaint about it. The prognostic features for Perthes disease mostly based on age
and x-ray appearance.
(ref: Apley’s Concise System of Orthopaedics and Fractures,
http://emedicine.medscape.com/article/826935-overview)

E. Present of Trendelenberg sign F


= Trendelenberg sign is an exaggerated up-and-down motion of the pelvis during ambulation
which the patient’s pelvis will sag. This sign usually present late on the physical examination.
The prognostic features depend on age of onset, involvement of femoral head, presence of
metaphyseal rarefaction and any lateral displacement of the femoral head. Therefore, this
answer is false.
(Ref: Apley’s Concise System of Orthopaedics and Fractures)

46. Concerning limb amputation:


A. True It is indicated in mangled limb.
- mangled limb/extremities are defined as high energy transfer or crush resulting in some
combination of injuries to artery, bone, tendon, nerve and/or soft tissue. It usually
caused by motor vehicle crash, auto-pedestrian crash, crush injury, farm/industrial
injury, fall from height and close range shotgun wound. It is not the absolute indication
for amputation as it is better delayed decision as we should consider attempting limb
salvage initially. The criteria to do immediate amputation are:
- Shredded muscle and transected nerves beyond elbow or knee especially posterior tibial
nerve in lower extremities.
- Crushed or mangled extremity with >6hours arterial occlusion upon arrival.
- Associated mangling or severe injury to the ipsilateral hand or foot.
- Severe associated polytrauma with persistent hypothermia, acidosis or coagulopathy
(“life over limb”).

(Ref:http://www.facs.org/trauma/publications/mangledextremity.pdf/AmericanCollegeOfSu
rgeon2002 /Management of the Mangled Extremities)

B. False In below knee amputations, the weight bearing is taken on the stump end.
In below knee amputations, they bear weight on patellar tendon, the lateral part of the
lower limb. Transmetatorsal, Symes and knee disarticulation all bears weight on stump end.
While, there should be no pressure on either the fibula head, tibial plate, hamstrings or end
of stump as this might severe pressure sore.
(Ref: www.rehabsa.co.za/content/articles/Amputation.pdf).

C. False The energy requirement to move the prosthesis is higher as the level of the
amputation is more distal.
The higher the level of a lower-limb amputation, the greater the energy expenditure that is
required for walking. As the level of the amputation moves proximally, the walking speed of the
individual decreases, and the oxygen consumption increases.
Impact of energy costs:
AMPUTATION LEVEL METABOLIC COST
TRAUMATIC VASCULAR
Trans tibial 25% 40%
Trans femoral 68% 100%
Bilateral trans tibial 41%
Trans tibial and trans femoral 118%
Bilateral trans femoral 186%
(Ref: http://emedicine.medscape.com/article/1232102-overview )

D. False Formation of neuroma will occur if the nerve is severed near the stump end
Any cut nerve will always form a tiny neuroma regardless of the site and occasionally this is
painful and tender. It can be manage by excising 3cm of the nerve above the bulb or
alternatively, the epineural sleeve of the nerve stump is freed from nerve fascicles for 5mm and
then sealed with a synthetic tissue adhesive or buried within muscle or bone away from
pressure points.
(Ref: Apley’s Concise System of Orthopaedics and Fractures)

E. True Phantom limb is a known complication following amputation


Phantom limb also one of the complications of amputation. It can be described as the feeling
that the amputated limb is still present. In most cases, the feeling recedes and eventually
disappears. A painful phantom limb is very difficult to treat. Other complications of amputation
are:
EARLY LATE
Secondary hemorrhage Skin – eczema, tender purulent lump, ulceration
Infection Muscle- risk of contracture,
Breakdown of the skin flaps Artery- lead to poor circulation
Gas gangrene Deep vein thrombosis
Neuroma
(ref: Apley’s Concise System of Orthopaedics and Fractures)

47. Ganglion
A. False Occurs most commonly on the anterior aspect of the wrist
The ubiquitous ganglion is seen most commonly on the back of the wrist.
(Ref: Apley’s Concise System of Orthopaedics and Fractures)

B. True Is due to cystic degeneration in the joint capsule or tendon sheath


it usually arises from cystic degeneration in the joint capsule or tendon sheath.
(ref: Apley’s Concise System of Orthopaedics and Fractures)
C. True Sometimes disappears after some months
The ganglion often disappears after some months, so there should no haste about the
treatment. It can be recurred again.
(ref: Apley’s Concise System of Orthopaedics and Fractures)
D. False Must be incised to prevent pressure necrosis of the underlying bone
Usually ganglion is characteristic by painless lump which well defined margin, cystic in nature
and non tender. Thus, it often disappeared after some months. The main complication of
ganglion is restricted movement of the wrist. It never causes pressure necrosis to the underlying
bone.
(Ref: Apley’s Concise System of Orthopaedics and Fractures,
http://archives.chennaionline.com/health/Homoeopathy/2005/04homoeopathy37.asp,
http://emedicine.medscape.com/article/1243454-treatment)
E. False On the dorsum of the hand is often tender
Patient with ganglion will present with a painless lump, usually on the back of the wrist. The
lump is well defined, cystic and non tender.
(ref: Apley’s Concise System of Orthopaedics and Fractures)

48. Injuries to the physis

A. False Commonly occur along the germinal zone of the growth plate

The injuries usually run transversely through the hyperthrophic (calcified) layer of the
growth plate, often veering off towards the shaft to include a triangular piece of the
metaphysis. This has little effect on the longitudinal growth, which takes place in the
germinal and proliferating layers of the physis. However, the fractures also can transverses
the cellular ‘reproductive’ layers of the plate.
(ref: Apley’s Concise System of Orthopaedics and Fractures)

B. True Are classifieds as Salter Harris Type 2 when fracture occurs through the physis and
metaphysis.

Salter Harris classification is widely used in physeal injuries. It can be divided into 5 types
which are
i. Type 1: Separation of the epiphysis. A transverse fracture through the
hyperthrophic of calcified zone of the plate. The growing zone of the physis is
usually not injured and growth disturbance uncommon.
ii. Type 2: Fracture through the physis and metaphysis. Similar to type 1 but towards
the edge the fractures deviates away from the physis and splits off the triangular
piece of metaphyseal bone. Growth usually not affected.
iii. Type 3: The fracture runs along the physis and then veers off the joint, splitting the
epiphysis. Inevitably it damages the reproductive zone of the physis and may result
in the growth disturbance.
iv. Type 4: Vertical fracture through the epiphysis and the adjacent metaphysic. This
fracture liable to displacement and a consequent misfit between the separated parts
of the physis, resulting in asymmetrical growth.
v. Type 5: Crushing of the physis without visible fracture. A longitudinal compression
injury of the physis. May result in growth arrest.
(ref: Apley’s Concise System of Orthopaedics and Fractures).

C. False Is associated with haemathrosis

Patients typically complain of what seems to be localized joint pain, often following a
traumatic event (eg, fall, collision). Swelling near a joint with focal tenderness over the
physis is usually present, as seen in the image below. Lower extremity injuries present as
an inability to bear weight on the injured side; upper extremity injuries present with
complaints of impaired function and reduced range of motion, quite similar to
ligamentous injury. Ligamentous laxity tests of the joints of the injured side may elicit
pain and positive findings similar to those indicative of joint injury. (An SH III or SH IV
fracture of the distal femur is the classic example.) Do not dismiss positive joint laxity
test findings as only involving the related joint tissues.
(ref: Apley’s Concise System of Orthopaedics and Fractures/
http://emedicine.medscape.com/article/1260663-overview).

D. False Do not warrant treatment via open reduction and internal fixation.

In displaced fractures Salter Harris Type 3 and 4, it demands perfect anatomical reduction. It
can be achieved by gentle manipulation under general anesthesia. If this successful, the limb
is held in a cast for 4-8 weeks (the longer periods for Type 4 injuries). Then, check x-rays at
about 4 and 10 days are essential to ensure the position has been retained. If the injury
cannot be reduced accurately by closed manipulation, immediate open reduction and
internal fixation is called for. The limb is then splinted for 4-6 weeks, but it takes that long
again before the child is ready to resume unrestricted activities.
(ref: Apley’s Concise System of Orthopaedics and Fractures)

E. True May result in premature fusion of the growth plate


The complications of injury to the physis are:
a. Premature fusion:
i. Type 1 and 2 injuries usually have excellent prognosis and the bone growth
is not adversely affected if properly reduced. Exceptions to this rule are
injuries involving the distal femoral and proximal tibial physes; both are
undulating in shape (transverse fracture may pass through several zones in
the physis and result in a focal point of fusion). Type 3 and 4 injuries more
likely to cause premature fusion of part of the growth plate, resulting in
cessation of growth or asymmetrical growth and deformity of the bone end.
b. Deformity :
i. Asymmetrical growth
ii. From malunion of adisplaced fracture (e.g.: a valgus elbow due to
proximal displacement or non union of a lateral humeral condylar
fracture).
(ref: Apley’s Concise System of Orthopaedics and Fractures).

49. Features suggesting tuberculosis in joint are:


- Long history – because it is such a chronic illness
- Involvement of only one joint
- Marked synovial thickening
- Marked muscle wasting
- Periarticular osteoporosis (X-ray)
(Ref: Apley’s Concise System of Orthopaedics & Fractures)
Other than that, the clinical features also can be of joint pain, muscle spasm and restriction of
movements and deformity to the affected joint. (Ref: Indian Journal of Orthopaedics)
75% of active tuberculosis cases come from the lung. In the other 25% of active cases, the
infection moves from the lungs, causing other kinds of TB, collectively denoted extrapulmonary
tuberculosis. This occurs more commonly in immunosuppressed persons and young children.
Extrapulmonary infection sites include the pleura in tuberculosis pleurisy, the central nervous
system in meningitis, the lymphatic system in scrofula of the neck, the genitourinar
system in urogenital tuberculosis, and bones and joints in Pott's disease of the spine. An
especially serious form is disseminated TB, more commonly known as miliary tuberculosis.
Extrapulmonary TB may co-exist with pulmonary TB as well. (Ref: www.wikipedia.com).
Bone and Joint tuberculosis results from haematogenous spread from a pulmonary or other
visceral or lymph node focus. (Ref: Indian Journal of Orthopaedics)
A. FALSE
B. FALSE
C. TRUE
D. TRUE
E. TRUE

50. De Quervain’s Disease


- De Quervain’s Disease is painful inflammation of tendons in the thumb that extend to the
wrist (tenosynovitis). Basically, what happens is the inflamed tendon and its coverings rub
against the narrow tunnel which they pass. It will cause acute tenderness at very tip of the
radial styloid. The tendons that affected are abductor pollicis longus and extensor pollicis
brevis because both tendons are tightly secured against the radial styloid by overlying
extensor retinaculum. (Ref: www.emedicine.com)
- The most common test to diagnose De Quervain’s disease is Finklestein’s test. The thumb is
tucked in close to the palm, and then turn the wrist sharply towards the ulnar side gives stab
pain over the radial styloid. When repeating movement with the thumb left free is relatively
painless. Thus, abduction of the thumb cause painless. Adduction the thumb which includes
cross the palm cause painful. In resistant cases need operation, which consist of slitting the
thickened tendon sheath. (Ref: Apley’s Concise System of Orthopaedics & Fractures)
A. TRUE
B. FASLE
C. FLASE
D. TRUE
E. TRUE

51. Complications of plaster immobilization (POP) are:


Local complication
- Tight cast will lead to limb swelling, pain and vascular compression. (Ref: Apley’s Concise
System of Orthopaedics & Fractures)
- Loose cast - Because swelling occurs with most fractures especially after reduction, padding
will be put under the cast to protect the skin. Once this padding gets compressed. After 48
hours when the edema is subsiding, the cast may be too loose to hold the bone ends in
position. This complication may seriously delay wound healing and may produce permanent
deformity. (Ref: Apley’s Concise System of Orthopaedics & Fractures)
- Nerve damage - Loss of power, tingling and numbness distal to the cast are signs of impaired
nerve function. The cause may be direct compression by bone ends or plaster pressure,
indirect compression of edematous tissue or tourniquet effect, or reduced blood flow.
- Pressure sores over the bony prominence (patella, heel, elbow, and head of the ulna). (Ref:
Apley’s Concise System of Orthopaedics & Fractures)
- Skin abrasion or laceration especially during removal of plasters using electric saw. (Ref:
Apley’s Concise System of Orthopaedics & Fractures)
Systemic complication
- The most serious is deep venous thrombosis leading to pulmonary embolism. Pain in the
calf is an important sign needing medical advice. (Ref: www.broadspine.com)

- Immobilisation in trunk plasters or plaster beds may also produce nausea, abdominal
muscle cramps, retention of urine and abdominal distention. (Ref: www.broadspine.com)

- Good nursing and diet with regular exercises will help ensure that the initial period of
extensive immobilization is achieved without complications. (Ref: www.broadspine.com)
*Non union is the complication for internal fixation. (Ref: Apley’s Concise System of
Orthopaedics & Fractures)
*Plaster of Paris in a type of cast that is used for immobilization of limbs. In case of patient with
allergy to the Plaster of Paris, therefore, they came out with another type of cast which is fiber
glass. But of course there are pros and cons of using it. Usually, patient with allergy of POP, they
will complaint of rashes and itchiness at the affected site. (Ref: I have asked doctor during
bedside teaching)
A. TRUE
B. TRUE
C. FALSE
D. TRUE
E. TRUE

52. Injuries to the spine


Fracture of the pedicle C2 is known as Hangman’s fracture
A. FALSE. (Ref: Apley’s Concise System of Orthopaedics & Fractures)

Anterior wedge compression fractures will affect the anterior part of the vertebrae only.
B. FALSE. (Ref: Apley’s Concise System of Orthopaedics & Fractures)
In fracture dislocation, the posterior ligaments are ruptured and the spine is potentially
unstable. If it is associated with greater degrees of displacement >25%, spine definitely
unstable and cord damage is likely.
C. TRUE. (Ref: Apley’s Concise System of Orthopaedics & Fractures)
Thoracolumbar junction injuries are sustained in a fall from height and combination of forces
due to axial compression and flexion.
D. TRUE. (Ref: Apley’s Concise System of Orthopaedics & Fractures)
Injury to spine can be either stable or unstable and complete or incomplete. Usually, as far
as the injury is stable and won’t affect the nerve roots and vertebral column, there will be no
neurological deficit. However, most of the spinal injury will affect the cord and cause
neurological deficits (Sorry. I can’t find reference for this question)

53. Acute lumbar disc prolapse

A. TRUE
A prolapsed (slipped) disc is a problem where the inter-vertebral disc is forced out of the
annulus fibrosus (the outer covering of the disc) due to mechanical forces increasing
intradiscal pressure. majority occur at the lowest 2 levels of spine ( L4/5 & L5/S1)
B. TRUE
C. TRUE
-The pain of may fluctuate from mild to severe. It can radiate to the legs due to pressure on
the sciatic nerve which supplies the leg muscles (Sciatica is severe pain referred to lower
limb happen due to compression to dural envelope of the nerve root).
-Distension of the annulus produces pain. The outer parts of the annulus are rich in nerves. If
the inner pulp tracks from within to the peripheral parts of the disc, stretching of the
annulus produces pain. The disc usually prolapses backward and to the side, left or right.
-The pain is described variously as aching or needle-like pricks, or burning, or like an electric
shock. Numbness and tingling may also occur in the same region.
(http://www.krishnaraman.com/Lumbardiscprolapse.pdf)
D. FALSE
-Plain x-rays – these are usually taken to rule out any fracture or malalignment. Dynamic x-
rays taken in flexion and extension may be performed to look for any instability. Plain x-rays
do not give any information on nerve root or spinal cord compression.
-CT L-spine – It gives some information on bony alignment but often fails to demonstrate a
disc prolapse. Occasionally it is combined with a myelogram to demonstrate any functional
compression/obstruction.
-MRI lumbar-spine – this is the gold standard in looking for lumbar disc prolapses and to
grading the degree of nerve root or cauda equine compression.
(http://www.vbsc.org.au/downloads/C_LumbarDiscPro_MM.pdf)
E. TRUE
-treatment for PID: rest, reduction and removal .
The goals of therapy are to reduce pain and inflammation by giving NSAIDs (ibuprofen
,naproxen).
-Surgical emergency – presence of cauda equina compression syndrome > 6 H because
cauda equina damage may be irreversible . symptoms may: Numbness around the bottom
and anus, Impotence or sexual dysfunction, loss of bowel or bladder control.
(Apley’s Concise Orthopedic, 3rd Edi, page 195-198)
(http://www.vbsc.org.au/downloads/C_LumbarDiscPro_MM.pdf)

54. Congenital Talipes Equino Varus (CTEV)

A. FALSE
The true etiology of congenital clubfoot is unknown. Most infants who have clubfoot have
no identifiable genetic, syndromal, or extrinsic cause.
B. FALSE
The male-to-female ratio is 2:1.
C. TRUE
-bilateral in one-third of cases (30-50% of cases.)
-Treatment usually surgery. No specific contraindications to surgery exist, although the
child's size that surgery is best performed at approximately age 6 months.
D. FALSE
-deformities are (1) equinus heel ( pointing downward) (2), varus hindfoot (tilted towards the
midline), (3) adducted & supinated forefoot
-The heel is small and empty. The heel feels soft to the touch .
-Similar deformities are seen with myelomeningocele and arthrogryposis ( always examine TRO
other causes!)
(Apley’s Concise Orthopedic, 3rd Edi, page 241)
(http://www.tsrhc.org/downloads/PDF/Clubfoot.pdf)
(e-medicine/club foot)

55. Tredelenburg sign is positive in this condition

A. TRUE
-The Trendelenburg test is a simple maneuver to evaluate the strength of the gluteus medius
and gluteus minimus muscle
-pain & shortening happen commonly at osteoporosis patient. Garden’s Classification of #
(1-incomplete #, 2-complete but still in line, 3- complete but displaced not in line, 4-
complete # with full displacement)
B. TRUE
-Poliomyelitis is a viral disease that can affect nerves and can lead to partial or full paralysis.
Clinical poliomyelitis affects the central nervous system (brain and spinal cord), and is
divided into nonparalytic and paralytic forms. It is characterized by asymmetric paralysis that
most often involves the legs. Bulbar polio leads to weakness of muscles innervated by
cranial nerves.
-hip abductor muscles consists of gluteus medius, gluteus minimus, tensor fascia lata with
superior gluteal nerve supply
C. TRUE
Positive sign in dislocation & subluxation of the hip
D. FALSE
E. TRUE
any painful disorder of the hip( synovitis, Tb, osteomyelitis, arthritis, RA)
(Apley’s Concise Orthopedic, 3rd Edi, page 202)

56. Foot disorder in Diabetes mellitus

A. FALSE
MTP joints hyperextended & IPJ flexed due to weak intrinsic muscles.
-(A claw toe is a lesser toe with dorsiflexion of the proximal phalanx on the lesser
metatarsophalangeal (MTP) joint and concurrent flexion of the proximal interphalangeal
(PIP) and distal interphalangeal (DIP) joints)
B. FALSE
-Dry gangrene can be left to demarcate before amputation, but wet gangrene & infection
are immediate amputation
C. TRUE
Neuropathic ulcer develop at areas of high plantar pressures (metatarsal heads, plantar
aspect of the great toe, heel or over bony prominences ), painless, unless they are
complicated by infection.
The gangrenous area is black, swollen and smelly. There is callus formation at the borders of
the ulcer. Its base is red, with a healthy granular appearance.
D. TRUE
-Ankle brachial systolic pressure index (ABSI) essential to measure perfusion to the muscle
tissue (usually need >0.8 before amputation for healing prosess)
E. TRUE
-Charcot foot (neuropathic osteoarthropathy) is a progressive condition characterized by
joint dislocation, pathologic fractures, and severe destruction of the pedal architecture.
-patient experience on lack of position sense & pain sensation make patient complaint of
instability, sweeling and deformity without warmth and tenderness)
(PDF Atlas of Diabetic Foot & Apley’s page 248)

57. Median nerve palsy

A T Thumb opposition and abduction are weak.


Median nerve supply opponens policis that fx to oppose(flex/ abduct)
the thumb. ( Netter’s concise orthopedic anatomy, 2nd edition, page207)
B F Median supply sensation at RADIAL three and a half fingers (palmar aspect)
Ulnar nerve supply ULNAR one and half fingers (palmar n dorsal)
Radial supply at RADIAL three and a half fingers (dorsal aspect)- typical area of sensory
loss- a small patch of sensory loss on the back of the hand at the base of the thumb.
(Netter’s concise orthopedic anatomy, 2nd edition, page204)
C T Low lesion is due to cuts in front the wrist or carpal dislocation.
High lesion due to forearm fracture or elbow dislocation.
Stabs and gun shot may damage the nerve at any level.
(Apley, 3rd edition, page 110)

D/E F/T In high lesion long flexors to the thumb, index (and middle )fingers are paralysed.
Showing pointing index sign.
P/S: Basicly in median nerve palsy, thenar eminence is wasted, the thumb abduction
and position is weak, sensation loss over radial 3rd n half digit, trophic changes may be
seen...+++ pointing index sign in high lesion.
(Apley, 3rd edition, page 118)

58. Degenerative spine disease.

A T Spinal stenosis, refers to the narrowing of the spinal canal anywhere along its axis.
Although the disorder often results from acquired degenerative changes (spondylosis) it
may also be congenital in nature.. The canal components that contribute to acquired
degenerative stenosis include the facets (hypertrophy, arthropathy), ligamentum
flavum (hypertrophy), posterior longitudinal ligament (OPLL), vertebral body (Bone
spur), the intervertebral disk, and the epidural fat. Spinal stenosis is most common in
the cervical and lumbar areas
LATERAL canal stenosis at any region of the spine may lead to nerve root compression.
The patients may experience radicular pain, weakness, and numbness along the
distribution of the affected spinal nerve. Lateral recess syndrome in the lumbar spine is
a result of such focal stenosis.
Stenosis of the CENTAL cervical and thoracic spine may result in myelopathy from cord
compression
Myelopathy
is the gradual loss of nerve function caused by disorders of the spine. Myelopathy can be
directly caused by spinal injury resulting in either reduced sensation or paralysis.
Degenerative disease may also cause this condition, with varied degrees of loss in
sensation and movement.
Spinal cord injury that results in myelopathy is classed as complete or incomplete. The
cord does not have to be severed to produce myelopathy. Significant damage to the spine
can cause complete paralysis or incomplete paralysis.
Complete myelopathy describes a spinal injury which results in no sensation below the
origin of the spinal injury. For example, a person with a spinal injury slightly above the
waist would not feel his or her legs, could not walk, would have loss of bladder control
and bowel function, and would not have sexual function. This is termed complete
because nothing below the injury works. In incomplete myelopathy as a result of spinal
injury, considering the same type of injury as above helps explain the distinction. A
person in this case might have bladder, bowel, and sexual function, but still not be able to
walk. In this type of myelopathy, some functions below the spinal injury may be
unaffected or only partially affected.
(http://www.wisegeek.com/what-is-myelopathy.htm)
CATEGORIES
Transverse lesion syndrome:
- corticospinal, spinothalamic, and posterior cord tracts are involved equally;
- associated with the longest duration of symptoms
- may represent end stage of the disease
Motor system syndrome:
- corticospinal tracts and anterior horn cells are injured causing spasticity;
- Central cord syndrome:
- motor and sensory deficits affected the upper extremities more severely than the
lower extremities;
- Brown-Séquard syndrome:
- ipsilateral motor deficits with contralateral sensory deficits
may be the least advanced form of the disease;
Brachialgia and cord syndrome: radicular pain in the upper extremity along with motor
and/or sensory long-tract signs.
(http://www.wheelessonline.com/ortho/myelopathy)

B F Babinski's sign may not be present until myelopathy becomes severe


(http://www.wheelessonline.com/ortho/myelopathy)
C T The symptoms come on after standing upright or walking for 5 to 10 minutes and are
consistently relieve by sitting or squatting with the spine somewhat flexed ~ spinal
claudication
(emedicine orthopaedics - spinal stenosis.)

D F Treatment can be conservative or surgical. The modes of conservative therapy include


rest, physical therapy with strengthening exercises for paraspinal
musculature, bracing, use of optimal postural biomechanics, nonsteroidal anti-
inflammatory medications, analgesics, and antispasmodics.
Surgical decompression is indicated in persons who experience incapacitating pain,
claudication, neurologic deficit, or myelopathy
(emedicine orthopaedics - spinal stenosis.)

E T The pathophysiology of spinal stenosis is related to cord dysfunction elicited by a


combination of mechanical compression and degenerative instability. With aging, the
intervertebral disk degenerates and collapses, leading to spur formation
(emedicine orthopaedics - spinal stenosis.)

59. Indication of internal fixation

A F We can do internal fixation in Gustilo type one – Dr Omar, other do External


Fixation.
Gustilo Anderson Classification of open fracture:
 I Low energy, wound less than 1 cm, clean
 II Wound greater than 1 cm with moderate soft tissue damage n contamination
 III High energy wound greater than 1 cm with extensive soft tissue damage n
contamination.
o IIIA Adequate soft tissue cover
o IIIB Inadequate soft tissue cover + periosteal stripping
o IIIC Associated with arterial injury
B F (NO FRACTURE x yah lah wat internal fixation), internal fixation is one of the principal
treatment for fracture NOT wound..

C/D T Indication of internal fixation:


1. Fractures that cannot be reduce except by operation.
2. Fractures that are inherent unstable and prone to redisplacement after
reduction
3. # that unite poorly and slowly- principally # of the femoral neck.
4. Phatological # in which bone disease may prevent healing.
5. Multiple fractures where early fixation reduce the risk of general complication.
6. # in patients who present severe nur sing difficulties
(Apley, 3rd edition, page 256)
E F this actually included in the risk of infection of internal fixation not indication.
(Apley, 3rd edition, page 256)

60. Compartment syndrome…

A T Bleeding and edema (inflammation/infection) can cause increase in pressure within


one osteofascial compartment.
(Apley, 3rd edition, page 294-295)
B T REMEMBER!!! That the presence of pulse does not exclude the diagnosis!!!
(Apley, 3rd edition, page 294-295)
C T Classic features of ischemia: pain, paraesthesia, pallor, paralysis and pulselessness.
(Apley, 3rd edition, page 294-295)
D F The limb should be nursed flat!!! Elevating the limb cause futher decrease in end
capillary pressure and aggravates the muscle ischemia.
Treatment:
 Cast, bandage and dressing must be completely removed.
 If the differential pressure (diastolic pressure- compartment pressure) less than
30 mmHg (4 kPa), immediate open fasciotomy is performed!!!!~ measure by
catheter that been introduced into the compartment!!
 If no facilities to measure, desicion to operate is to be made on clinical ground.
– limb should be examine 15 min interval, if no improvement within 2 hours-
perform fasciotomy!!! (muscle dead after 4-6 hours of total ischemia.)
 After fasciotomy, wound left OPEN, inspect 2 days later ~ if muscle necrosis-
debride!!! , if healthy- suture (without tension), or skin grafted or simply
allowed to heal by secondary intention!!!! EASY kan???? ; ).
(Apley, 3rd edition, page 294-295)
E T Nerve is capable of regeneration but muscle once infracted can never recover
and is replaced by inelastic fibrous tissue (Volkmann’s ischemic contracture).
(Apley, 3rd edition, page 294-295)

61.

Q ANS REASON(S) SOURCE


rd
A F Galeazzi #=# of distal 1/3 of radius + dislocation of Apley’s 3 ed, page 323
DRUJ
B T 1)PIN maps around the radial neck, hence # of radial 1)http://emedicine.medscape.co
head or displaced # thru the radial neck tend to injure m/article/1240337-overview
the PIN.
2)pan arab-journals, volume 2:
2)Pure motor lesion leads to no sensory disturbances. radial nerve compression
It may be damaged in fracture of the proximal end of syndromes
the radius or during dislocation of the radial head.
Extra notes: # of radial neck is commoner in
paeds age group. Proximal epiphysis is
cartilaginous, therefore prone to # rather
than radial head which has a hard articular
surface).

C T We retrospectively reviewed the results of operative Chronic Monteggia lesions in


treatment of chronic Monteggia lesions (Bado type I children. Complications and
or the equivalent) with anterior radiocapitellar results of reconstruction. J Bone
dislocation in seven patients. There were fourteen Joint Surg Am. 1996
complications, including malunion of the ulnar shaft Sep;78(9):1322-9
in one patient; residual radiocapitellar subluxation in
two patients (one anterior and one posterolateral);
radiocapitellar dislocation (dynamic anterior
subluxation of the radial head in supination) in one
patient; transient ulnar-nerve palsy in three patients
(with residual weakness in two); partial laceration of
the radial nerve in one patient; loss of the fixation in
two patients; and non-union of the ulnar osteotomy
site, compartment syndrome, conversion reaction,
and possible fibrous synostosis of the forearm in one
patient each. The patients lost a mean of 36 degrees
of pronation and a mean of 27 degrees of supination
of the forearm compared with the contralateral,
uninjured extremity.
D T 1)greater threat n difficult dx if the forearm is 1)Apley’s 3rd ed page 322
wrapped up in plaster

2)Dr Fairudz also said that # of the femur rarely give 2)Dr. Fairudz
rise to compartment syndrome coz there is large
space there. But # of the tibia n forearm tend to result
in compartment syndrome due to lack of space for
oedema to expand.

E T radial # tend to displaced coz of strong muscle Apley’s 3rd ed page 322
contraction. Therefore, need internal fixation.
Positioning of the hand depends on level of #: upper
1/3=supinated, middle 1/=neutral position, lower
1/3=pronated.

62. Colles fracture

Q ANS REASON(S) SOURCE


A T 1)Distal radius fractures are classified as intra-articular 1)Christopher H. Allan, MD.
if the fracture line extends to or through the articular Orthopaedics Knowledge Online,
surface of either the radiocarpal or the distal AAOS).
radioulnar joints

2)Examples of intra-articular distal radial # are distal 2)Wheeless online textbook of


Barton # and volar Barton # orthopaedics
B F 1)Fracture that Colles described (Abraham Colles was 1)Essential orthopaedics and
a professor of surgery in Dublin): trauma, Dandy and Edwards, 4th
- was within 1 inch of the wrist joint ed page 205.
- had dorsal angulation of the distal fragment
-had dorsal displacement of the fragment
- was a/w a fracture of the ulnar styloid
ANS: false as dorsal displacement is one of the criteria

C T usual case: postmenopausal woman with hx of FOOSH Apley’s 3rd ed page 324.
(fall on outstretched hand).
D F treatment option: 1) undisplaced: dorsal splint until Apley’s 3rd ed page 324-325.
swelling subside, the cast for 4 weeks. 2) displaced:
reduction under anaesthesia. Then, dorsal backslab
(below elbow to metacarpal neck). C) comminuted &
unstable: PC K-wire, or external fixation. Even the
unstable # pun x perlukan ORIF.
E F 1)Gunstock deformity aka cubitus varus is a cx of 1)Apley’s 3rd ed page 313.
supracondylar # in children. So called gunstock coz its
varus shape resemble gunstock (butt end of the
shotgun).

2)Malunion in Colles # is common d/t incomplete 2)Apley’s 3rd ed page 325


reduction or overlooked displacement within within
the plaster. Treatment isn’t necessary, if got marked
disability then radial osteotomy can be done.

63. Regarding Tuberculosis, which statement is true:-

Q ANS REASON(S) SOURCE


A T Pathophysiology: 1) hematogenous spread. 2) thru http://boneandspine.com/arthri
OM metaphysis (in neonates) tis/tuberculous-
Pathophysiology 1): bac lodge in synovial tissue arthritispathology-clinical-
leading to synovitis > synovial membrane secretes features/
excessive fluid > later, proliferation, thickening and
studding of its inner surface with tubercles. On its
outer surface, fibrosis develops. > tuberculous
granulation tissue covers the hyaline articular
cartilage as a pannus > eventually, destruction of
underlying articular cartilage and subchondral bone
occur. > as the dss progress, increasing amount of
caseous necrotic material and tuberculous exudates
are released > “pus” forms > spreads by dissecting
along tissue planes b/w muscles or muscle sheaths,
being limited by the deep fascia. > With increasing
tension the deep fascia perforates and the abscess
becomes subcutaneous (eg: psoas abscess). > If the
original focus remains active and these abscesses
remain untreated, they will rupture externally
through the skin to form sinuses, the results being the
inevitable secondary infection by pyogenic bacteria,
and complete destruction of the affected joint.
2)OM of metaphysis can lead to infective arthritis in
neonates coz their metaphysis is intracapsular 2) WHEELESS ONLINE TEXTBOOK
(especially metaphysis of shoulder, hip, radial head, OF ORTHOPAEDICS
and ankle). Then, the pathophysiology follow the one
that has been explained above.
B T The affected joint will be stiff, and soon the “night- http://boneandspine.com/arthri
cries” develop; because irritation from the process is tis/tuberculous-
low-grade, muscle spasm protects the part quite arthritispathology-clinical-
satisfactorily during the day, but when the child is features/
asleep the protective action of the muscles is lost, and
on motion, pain is produced; hence, the cry.
C T as explained above.
Can also look at Apley’s 3rd ed page 24
D F anti-Tb chemotherapy (9 months course  first 2/12 WHO guidelines on treatment of
of HRZE and the next 7/12 of HR. if worse, then tuberculosis, 4th ed, page 88 and
arthrodesis. page 32: table 3.2a

64. Regarding ulnar Nerve:-

Q ANS REASON(S) SOURCE


A F Less clawed in higher lesion. Ulnar ALL ARE TAKEN FROM DR
claw=hyperextension of the 4th and 5th finger at MCP OMAR’S NOTES AND BASED ON
joint, but flexed at IP joint. (need good knowledge of HIS EXPLANATION DURING
anatomy of ulnar nerve and the muscles that it LECTURE. ANY DOUBT CAN JUZ
supplied to understand this question :p) READ ON ANATOMY AND THEN
 Clawing occur in lower lesion (usually at the ONLY WE CAN U/STAND THE
level of the wrist) coz the long flexor that is TOPIC WELL
innervated by ulnar nerve (Flexor Digitorum
Profundus) is not affected  hence, flexion of
IP joint at 4th and 5th finger. Ulnar nerve lesion
at this site paralyzed the lumbricals muscle
which flexed the MCP joint  therefore the
4th and 5th fingers are extended at their MCP
joint. Paralysis of dorsal interossei and palmar
interossei makes abduction and adduction of
the fingers impossible.
 In higher lesion (usually at the cubital tunnel),
both the FDP and INTRINSIC MUSCLES
(lumbricals, dorsal interossei, palmar
interossei) are affected. The affected hand
will show extension of MCP joint but no
clawing noted coz FDP no longer working.

B F Sensory part of ulnar nerve supplies ulnar 1 and a half


finger (i.e the little finger + half of ring finger).
C T Adduction of thumb is due to the action of Adductor
Pollicis. Paralysis of Adductor Pollicis is tested by
Froment’s sign=give the pt a piece of paper to grasp
and examiner will try to pull the paper. In normal ppl,
they will be able to maintain the paper firmly. If the pt
got paralysis of Adductor Pollicis, we will see the IP
joint of the thumb flexed in order to maintain the
grasp. This is merely compensation by the FPL (flexor
pollicis longus which is innervated by AIN of median
nerve).
D T Lumbricals muscle’s action is to flex the MCP joint. In
ulnar nerve lesion, MCP joint is extended due to
paralysis of lumbricals (intrinsic muscles of the hand).

65. Ankylosing Sponylitits


A. Associated with HLABRA 7 (F)
HLA-B27 (Apley’s concise pg.30)
B. Always associated with false positive rheumatoid factor (F)?
C. Common in Africans (T)
Prevalence is about 0.2 per cent in Western Europe, but much lower in Japanese and
Negroid peoples (Apley’s concise pg.30)

66. In stenosing tenovaginitis (trigger finger)


A. The extensor tendon is commonly involved (T)
The 1st dorsal compartment (abductor pollicis longus and extensor pollicis brevis) and the
second dorsal compartment (extensor carpi radialis longus and brevis) are the one most
commonly affected (Apley’s pg.165)
B. The usual cause is thickening of the fibrous tendon sheath (T)
Over-use or repetitive minor trauma will result in synovial inflammation which causes the
secondary thickening of the sheath and stenosis of compartment that further
compromises the tendon. (Apley’s pg.165)
C. The triggering occurs during flexion of the involved finger (T)
Signs of TF are as follows:
 Triggering on active or passive extension by the patient
 Palpable snapping sensation or crepitus over the A1 pulley
 Tenderness over the A1 pulley
 Palpable nodule in the line of the (flexor digitorum superficialis)FDS, just distal to the
MCP joint in the palm
 Fixed-flexion deformity in late presentations, especially in the proximal
interphalangeal (PIP) joint
 Evidence of associated conditions (eg, RA, gout)
 Early signs of triggering in other digits (may be bilateral)
(http://emedicine.medscape.com/article/1244693-overview)
D. A tender nodule can be felt in front of the affected area (T)
E. The ring and middle fingers are most commonly affected (F)
Volar flexor tenosynovitis (ie, trigger finger)
 This type of tenosynovitis most commonly affects the thumb or ring finger.
 Most common in middle-aged women
 More common in patients with diabetes
 Locking of involved finger in flexion is followed by sudden release (hence the name
trigger finger); hand pain radiates to fingers. In more severe cases, the finger may
require passive manipulation to regain extension.
(http://emedicine.medscape.com/article/809777-overview)
67. Gas gangrene:
A. Is caused by Streptococcus pyogenes infection (F)
Organisms in the spore-forming clostridial species, including Clostridium
perfringens, Clostridium septicum,and Clostridium novyi, cause most of the cases
B. Is characterized by myonecrosis (T)
The hallmarks of this disease are rapid onset of myonecrosis with muscle swelling,
severe pain, gas production, and sepsis.
C. Often manifests itself within 24 hours of injury (T)
The incubation period is usually less than 24 hours but has been described to be
anywhere from 7 hours to 6 weeks, though when symptoms start, clinical
deterioration can occur within hours.
D. Causes little pyrexia but increases pulse rate (F)
Vital signs: Unusually, fever is not a prominent feature of infection and may only
be low grade throughout the clinical course. The degree of systemic involvement
may produce a spectrum of changes from tachycardia through outright septic
shock including hypotension and diaphoresis.
E. Is treated mainly by strong antibiotics (F)
The treatment is a combination of antibiotics, surgery, and hyperbaric oxygen

(All answers from http://emedicine.medscape.com/article/809777-overview)

68. Rotator cuff tears:


A. May occur as a complication of chronic tendinitis (T)
One of the signs of chronic tendinitis is crepitus/clicking during movement that
suggests a partial tear of the rotator cuff (Apley’s pg.145)
B. Is mostly presentable as limitation of glenohumeral joint movement in all directions (F)
Basically partial/complete tear will result in weakness in abduction. To distinguish
between partial and complete tears, pain is abolished by injecting a local
anaesthetic and if active abduction is now possible, the tear must be only
partial.(Apley’s pg.146)
C. Is associated with ‘hook’ shaped acromion (T)
Hook shaped acromion is a Type III acromion which has more significant
association with rotator cuff tears compared to Type I acromion which has flat
surface(3% only assoc wt rotator cuff tears)(MRI of the Shoulder pg.120)
D. Is commonly occur to supraspinatus tendon (T)
Partial tears of the rotator cuff frequently occur with supraspinatus tendinitis;
indeed, it is possible that tendinitis is precipitated by a minor tear.(Apley’s pg 146)
E. Is ideally repair surgically in all elderly patients (F)
Operation is contraindicated in old or sedentary individuals, and long standing cases that
are painless and accompanied by satisfactory function. (Apley’s pg. 147)

69. Developmental dislocation of the hip


A. Can be demonstrated by Ortoloni test.(T)
How to do? Put your thumb at the medial thigh; fingers at greater trochanter à flex hip join
90˚à abduct
If normal à smooth abduction 90˚
If dislocate à the movement is impeded à sometimes if you press the greater trochanter; the
dislocation can be reduce
Other test: Barlow’s test
B. Trendelenburg test is positive in a child who is able to stand.(T)
Positive in 4 conditions:
i. Dislocation & subluxation of the hip
ii. Weakness f the abductors
iii. Shortening of the femoral head
iv. Painful hip

C. It is common in a child who presented with breech position during intrauterine. (T)
Because in breech position + extended leg would favour hip dislocation
Other causes:
- Genetic à generalized joint laxity & shallow acetabula
- Hormonal changes in late pregnancy à aggravate joint laxity
- Postnatal factors à the baby is carried with hips & knee fully extended
D. Boys are more common than girls. (F)
G>B ration 7:1 (Apley concise pg205)
E. Increase acetabular angle in plain x-ray.(T)
This is one of the features in plain x-ray. Normal angle: <30˚
Other 2 features:
i. Epiphysis should medial to a vertical line (Perkin’s line) below horizontal line
(Hilgenreiner’s line)
ii. With the hips abducted 45˚ the femoral shafts should point into the acetabula
(source: apley concise pg 206)

70. Regarding malignant bone tumours


A. Fibrosarcoma can arise from giant cell tumour (F)
No definite answer from the book
Btw fibrosarcoma originate from fibrous tissue; GCT is from marrow so maybe false
Really not sure la this one L
B. Ewing tumour presented with ‘onion peeling’ in plain x-ray. (T)
Everyone know!
C. Osteoblastic activity in prostate carcinoma (T)
Everyone know!

71. Management of diabetic foot ulcer (risk of amputation)


A. Superficial ulcer (F)
B. Uncontrolled hyperglycaemia (F)
C. Previous amputation (T)
D. Uncontrolled infection (T)
E. ABSI > 1.1 (F)
No definite answer
Try read this article maybe helpful: uitm library – online database – ebook – American
College of Surgeon – Ch 6 vascular surgery – diabetic foot

72. Radiological features of OA


A. Subchondral cyst (T)
B. Osteophyte formation (T)
C. Widening of joint space (F)
Should be narrowing
D. Ankylosis (F)
Not sure because no definite answer from the book.
Btw ankylosis is immobile joint d/t disease; arthtrodesis is surgically induced
Can we see on x ray such a mechanical thing???
E. Periosteal osteoporosis (F)
Not sure because no definite answer from the book
Btw periosteum is not the bone. Histologically it contain condense fibrous tissue (Weather’s
functional histology)
So, how come it can be osteoporotic???

73. Ganglion cyst around wrist:


A. Common in male
B. Present as solitary nodule only
C. Mucoid degeneration of collagen and connective tissue
D. Contain mucin, albumin and globulin
E. Common in volar aspect of wrist

A little bit to know about ganglion cyst:


What is it? – tumor or swelling arise from cystic degeneration in the joint capsule/tendon
sheath.

A. It is more common in women


(ref:http://www.emedicinehealth.com/ganglion_cyst/article_em.htm)
So, the answer is FALSE

B. Multiple small cysts can give the appearance of more than one cyst, but a common stalk
within the deeper tissue usually connects them
(Ref: http://www.emedicinehealth.com/ganglion_cyst/article_em.htm)
Patients with ganglion cysts typically have only one lesion, but some people seem to be
predisposed to having them in multiple locations.
Even the word ‘only’ pun slalu indicate FALSE in MCQ.
So, the answer is FALSE

C. The etiology is unknown, but the theory is there is cystic degeneration of the mucoid
connective tissue, specifically collagen, in the joint capsule/ tendon sheath that forming
the cyst ,when Ledderhose described it as such.
(Ref: http://emedicine.medscape.com/article/1243454-overview)
Thus, the answer is TRUE

D. It has also been suggested that degeneration of the connective tissue is caused by an
irritation or chronic damage causing the mesenchymal cells or fibroblasts to produce mucin
(fluid in the cyst). (http://emedicine.medscape.com/article/1243454-overview)
The mucin itself contain high concentration of hyaloronic acid, as well as glucosamine,
albumin and globulin.
(Ref:Google book: Essential of physical medicine and rehabilitation)
Thus, the answer is TRUE

E. Ganglion cysts can occur at any joint or tendon sheath, but they most often present in the
dorsum of the wrist at the scapholunate joint, (60-70% of all hand and wrist ganglia )
followed by the volar wrist (20%). (http://emedicine.medscape.com/article/1243454-
overview)
So, the answer is FALSE
74. In crystal deposition disorder

A. FALSE
Crystal deposition disorder bkn saje gout.. there are few other clinical condition assoc with
crystal deposits. Clinical conditions associated with crystal deposition d/o:
i. Gout (monosodium urate monohydrate crystal) –disorder of purine metabolism,
hyperuricaemia
ii. Tx principle in acute attack- resting the joint
a. -give large dose of NSAIDS
iii. Pseudogout (calcium pyrophosphate dihydrate),
iv. Calcium hydroxyapatite (HA) deposition disorder
(Ref: Apley’s concise system pg. 37-39)

B. TRUE
Yes, diagnosis rest on identifying the crystal in syn. fluid. Characteristics of crystal in gout
(monosodium urate) seen under a polarizing microscope, is bright yellow needlelike
negatively birefringent crystal.
(Ref: Cleveland Journal of Medicine- the gout diagnosis
http://www.ccjm.org/content/75/Suppl_5/S17.full.pdf )

C. FALSE

D. TRUE

E. TRUE
(Reasoning for C,D.E are all written dlm jwpan A)

75. Amputation

A. TRUE
In the presence of extensive wet gangrene of foot, a guillotine amputation through distal
tibia or fibula may be indicated.
(Ref: Essential of surgery: scientific principle and practice)

B. FALSE
Major weight bearing areas of below knee amputation  patella tendon and tibial flares
Symes amputation (through ankle), major weight bearing area end of stump

C. TRUE
Above knee amputation major weight bearing are= ischium
(Ref: Lower limb amputation for ischaemia with special reference to the diabetic patient-
CME article/paper)
D. TRUE
Advantage of above knee amputation is greater than 90% primary healing rate.
Disadvantages, however, are that only 40 to 50% of above knee amputation patients can
learn to ambulate independently. There is a large increased energy requirement amounting
to 80 to 120° greater than normal
(ref: http://www.vascdocs.com/health/amputation.shtml)
The increased energy requirements of prosthetic ambulation can limit the use of a
prosthesis. An individual who has a lower extremity amputation and requires a walker or
crutches to ambulate (with or without a prosthesis) uses 65% more energy than does
someone with a normal gait.

E. TRUE
Late complication of amputation: neuroma(swelling of nerve)-a cut nerve always forms a
tiny ‘neuroma’ and occasionally painful and tender
(Ref: Apley’s concise system: Amputation, page 33)

76. Regarding tenosynovitis:


A. In the proliferative type, it starts within the synovial lining of tendon sheath or invades the
tendon from involvement of a contagious joint.
B. In crystalline type, precipitation of crystalline outside the confines of an enclosed space
triggers fulminant inflammatory reaction.
C. Calcium pyrophosphate deposition disease can cause acute inflammatory tenosynovitis
within carpal tunnel.

81. DVT(pg 134,135)

A. FALSE
“Homan’s sign-increased calf pain on passive dorsiflexion of the foot and toes-is often thought to
secure or exclude the dx of DVT. This is regrettable as more accurate techniques have shown that
the sign is unreliable” Concise Apley’s pg 135

B. TRUE
The symptoms for DVT includes:
a. Asymptomtic;common
b. Symptomatic; pain in calf or thigh, swelling, soft-tissue tenderness, increased temp,
increased pulse rate.

C. The surgical management for DVT includes:

D. TRUE
The investigations of DVT are:
a. Apart from basic-line ix, venography and ultrasound should be done to confirm the dx.

E. TRUE
The medical management for DVT are:
a. Heparin IV, warfarin, low-molecular-weight heparin

Answers taken from Appley Concise


82. In slipped capital femoral epiphysis (pg 214, 215)

A. FALSE. Prevalence; Boys affected more than gurls


B. TRUE
“The onset may be sudden and in30 percent there is a history of trauma(acute slip)” pg 214
C. FALSE bcoz it occurs in the hypertrophic zone of the cartilaginous growth plate
D. FALSE bcoz of the word confine, restricted. It usually occurs in male patients age 15 to 16
but bcoz of the word restricted=only, it is FALSE

83. in fracture of phalanges and metaphalanges(not well-elaborated in Concise. I’ll still be looking
up for it and ill email later?)
A. Undisplaced # of phalanges can be splinted to its neighbor
B. It is important to correct malrotation
C. Bernett # occur at the base of 5th metacarpal
D. Immobilization should be at least 6/52
E. Stiffness is the most important complication

84. Regarding CTEV(club-foot) (pg 241, 242)

Clinical features of congenital talipes equinovarus;CTEV(idiopathic club-foot);(Concise Apley’s pg


241)
i. Foot; both turned and twisted inwards
ii. Hindfoot; varus(tilted towards the midline)
iii. Mid-foot and forefoot; adducted n supinated(twisted medially and the sole turned
upwards)
iv. Soles; faces posteromedially
v. Heel; small n high, equivinus(pointing downwards)
vi. Deep creases; posteromedially

Treatment (Concise Apley’s pg 241)


Conservative; manipulation, strapping or serial of casting
Operative; lizarov fixator
Kirschner wires
Dennis Browne boots(prevention of complications of surgery)
A. Hindfoot dorsiflex (FALSE) bcoz the c/fx for CTEV hindfoot is in varus
B. Forefoot is internally rotated(TRUE) its in varus position
C. Forefoot is everted(FALSE) bcoz its suppose to be inverted
D. Calf muscle is under-developed()
E. Txm begins 2-3 days after birth(FALSE) treatment starts conservatively 1-2 days after birth
but if it fails then proceed with surgical treatment(Concise Apley’s pg 241)

85. Regarding Amputation

A. F – because wt is taken on the stump when the amputation done through or near a joint, eg;
through knee or through ankle amputation. In below knee amputation or transtibial
amputation, the wt was transmitted to the patellar,knee.
B. T – wt can be transmitted through ischial tuberosity, patellar tendon, upper tibia or the soft
tissue. (Apley, pg 133)
C. T – energy requirement in above knee prostheses (transfemoral amputation)
Unilateral : 60-70%
Bilateral : >200%
(http://books.google.com.my/books?id=2sLvNV58V8oC&pg=PA24&lpg=PA24&dq=weight+tr
ansmission+in+below+knee+amputation&source=bl&ots=1LwvydVVo3&sig=tLcnCkr8J70EcVf
5D3zBRFBwlbI&hl=en&ei=Eg6MTfbuA8j5rAfmkv3sDQ&sa=X&oi=book_result&ct=result&res
num=3&ved=0CCkQ6AEwAg#v=onepage&q=weight%20transmission%20in%20below%20kn
ee%20amputation&f=true )
D. T – a cut nerve always forms a tiny neuroma n it is painful n tender (Apley, pg 133)

86. Fat embolism

A. T
B. T – no specific tx for fat embolism but the most important measure is to reduce hpoxemia
by giving O2.
C. T
D. F – mostly occur in young adult after closed fractures of long bones
E. T
(source: Apley 3rd edition, pg 264)

87. Archilles tendon rupture

A. T (Apley 3rd edition, pg 250)


B. T – Simmonds’ test: ask px to prone, squeeze the calf, if tendon intact-the foot will
plantarflex. If not, means the tendon is ruptured and the test is +ve. (Apley 3rd edition,
pg 250)
C. F – plaster can be applied if the x is seen early. (Apley 3rd edition, pg 250)
D. T - When the calf muscle contracts, it shortens and pulls on the Achilles tendon resulting
in pushing the foot downward. We use our Achilles tendon in this manner when pushing
off during walking, running, and jumping.This action of the Achilles is also what we use
to walk on our tip toes.
(http://physicaltherapy.about.com/od/humananatomy/p/AchillesTendon.htm)
E. T - In the past, the complications of surgical repair of the Achilles tendon made surgeons
think twice before suggesting surgery. The complications arose because the skin where
the incision must be made is thin and has a poor blood supply. This can lead to an
increase in the chance of the wound not healing and infection setting in. Now that this is
better recognized, the complication rate is lower and surgery is recommended more
often.

88. Carpal tunnel syndrome

A. F – more common in women


(http://www.ncbi.nlm.nih.gov/pubmedhealth/PMH0001469/)
B. F – in late case, causing thenar m/s wasting (Apley 3rd edition, pg 166)
C. T - In autoimmune diseases, the body's immune system abnormally attacks its own
tissue, causing widespread inflammation, which, in many cases, affects the carpal tunnel
of the hand. Such autoimmune diseases include rheumatoid arthritis, systemic lupus
erythematosus, and thyroiditis, which can lead to hypothyroidism..
(http://www.umm.edu/patiented/articles/what_causes_carpal_tunnel_syndrome_0000
34_3.htm)
D. T – Phalen’s test (http://www.ncbi.nlm.nih.gov/pubmedhealth/PMH0001469/ and
Apley, pg 166)
E. F – transverse carpal ligament (Apley,pg 166)
89. Regarding examination of the knee

A. The answer is false because anterior drawer test indicate ACL injury not PCL. PCL was tested
by posterior drawer test—physical examination in orthopedic surgery by Lee Joon
Kiong,page 130

B. Lachman test done when the patient’s knee is flexed 20 degree-Apley’s Concise System of
Orthopedic & Fracture,page 223
-the cruciate ligaments are tested with both knees flexed 90 degree

C. actually not really sure but you can look at this explanation:
- Q angle is the angle formed by a line drawn from the ASIS to central patella and a second
line drawn from central patella to tibial tubercle;
- an increased Q angle is a risk factor for patellar subluxation
- normally Q angle is 14 deg for males and 17 deg for females;
- Agliettis et. al. Clin. Ortho 1983:
- 75 normal males: Q angle = 14 deg (+/- 3)
- 75 normal females: Q angle = 17 deg (+/- 3)
- biomechanics of patellofemoral joint are effected by patellar tendon length & the Q
angle;
- q angle is increased by:
- genu valgum
- increased femoral anteversion
- external tibial torsion
- laterally positioned tibial tuberosity
- tight lateral retinaculum

- Clincal Determination: (see examination of the patellofemoral joint)


- deficiency of vastus medialis oblique is best assessed while leg is suspended in 15-20 deg
of flexion;
- large convexity at superomedial corner of patella indicates vastus medialis deficiency;
- this is best measured both w/ the knee in extension as well as flexion;
- Q angle may not be accurrate in extension, since a laterally dislocated patella may give
false impression that the Q angle is normal;
- in flexion, this is not a problem since the patella is well seated in the trochlear
groove;
- a sitting Q angle of more than 8 deg is abnormal;
- Wheeless' Textbook of Orthopaedics-
http://www.wheelessonline.com/ortho/q_angle_of_the_knee

D. There must be a moderate amount for this test to be positive. Too much fluid can prevent
the patella being pushed on to the condyles; too little will not life the patella free from
them- http://www.olympus.co.in/SingleColumn-Medical-V-11-01.pdf

E. The answer is false because the word ‘classically’ for bucket handle tear of knee menisci.
Mc Murray is actually test to check meniscus injury whether lateral or medial meniscus.
Bucket Handle Test can be discovered during straightening the femur, if patient has suffered
repeated locking of the joint. This indicates a lesion of cartilage in its anterior section. At the
same time, click can be produced from the same cartilage posteriorly
.-McMurray Test: (Br.J.Surg. 1942. 407)-
http://www.wheelessonline.com/ortho/mcmurray_test_brj_surg_1942_407
90. Posterior hip dislocation

A. posterior dislocation is the commonest- Apley’s Concise System of Orthopedic &


Fracture,page 361

B. leg is short and lies adducted, internally rotated & slightly flexed- Apley’s Concise System of
Orthopedic & Fracture,page 361

C. usually occur in road accident when people seated in a truck or car is thrown forward,
striking the knee against dashboard. Femur is thrust upward and femoral head is forced out
of its socket.- Apley’s Concise System of Orthopedic & Fracture,page 361

D. fracture of the femoral head prevent the reduction.


- Management of posterior hip fracture dislocation
- attempt closed reduction unless:
- bone fragment is noted in the acetabulum:
- it is essential to determine whether the hip is stable following reduction by stress testing;
- stress testing is especially important if posterior wall frx is present;
- type II frx of significant size may be treated non operatively if there is no posterior hip
instability;
- unstable reduction:
- hip redislocates w/ 90 deg flexion;
- frx of the femoral head prevents the reduction
- comminuted frx of acetabulum, esp. posterior wall frx.
- Wheeless' Textbook of Orthopaedics
http://www.wheelessonline.com/ortho/type_ii_v_posterior_fracture_dislocations

E. complication-sciatic nerve injury (10-20%), avascular necrosis(10%) and OA- Apley’s Concise
System of Orthopedic & Fracture,page 361

91. Acute lumbar disc prolapsed

A. two common level is L4/L5 and L5/S1- Apley’s Concise System of Orthopedic & Fracture,page
195
B. If it occurs at L4 and L5 intervetebral disc space, both L4 and L5 can be compressed. L4
compression occur in lateral herniation and for L5 compression occur in medial herniation-
Lecture Dr Omar,Degenerative Spine Disease, page 32
C. X-ray is essential to exclude bone disease. CT and MRI are the best way identifying the disc
and localizing the lesion- Apley’s Concise System of Orthopedic & Fracture,page 196
D. the symptom actually depend on structure involved & degree of compression:
-presure on ligament-backache
-Pressure of the dural envelope of the nerve root - severe pain referred to the lower limb
(sciatica)
-compression of nerve-numbness, paraesthesia & muscle weakness- Apley’s Concise System
of Orthopedic & Fracture,page 195

E. indication for operative removal of disc:


1) Cauda equine compression syndrome which does not clear up within 6 week of starting
bed rest & traction.
2) Persistent pain & severely limited straight-leg raising after 2 week of conservative
treatment.
3) Neurological deterioration while under conservative treatment
4) Frequently recurrent attack
-Apley’s Concise System of Orthopedic & Fracture,page 197

92. Spondylolisthesis
A. Spondylolisthesis means vertebral displacement. Actually, Normal laminae & facets act as
locking mechanism to prevent each vertebra from moving forward on the one below.
Forward shift occur when this mechanism fail. X-ray finding: forward shift of upper part of
spinal column on the vertebral below & elongation of the arch or defective facets may be
seen- Apley’s Concise System of Orthopedic & Fracture,page 198-199
B. listhesis nearly always between L4 and L5 or L5 and sacrum- Apley’s Concise System of
Orthopedic & Fracture,page 198
C. degenerative is one of the conditions. Look at the causes of it:
i. Dysplasia of lumbosacral facet joint
ii. Separation or stress fracture(lysis) through neural arch,allowing anterior part of
vertebrae to slip forward
iii. Osteoarthritic degeneration of facet joint(L4/L5)
iv. Destructive condition-TB,#,neoplasia
- Apley’s Concise System of Orthopedic & Fracture,page 198

D. in children, it is called dysplastic spondylolisthesis,usually painless- Apley’s Concise System of


Orthopedic & Fracture,page 198

E. indication for operation:


-at any age if the symptom disabling
-in the young adult even with moderate symptom
-neurological compression is marked
-Apley’s Concise System of Orthopedic & Fracture,page 198

93. Multiple myeloma

A. T –Multiple myeloma is a malignant B-cell lyphoproliferative disorder of the marrow


predominantly plasma cells ( Apley’s pg 93 )
B. F –Marrow-cell proliferation & increased osteoclastic activity result in osteoporosis
and appearance of discrete lytic lesions throughout the skeleton (myelomatosis)
( Apley’s pg 93 )
C. T – It is associated with features of the marrow-cell disorder which are plasma
protein abnormalities, increased blood viscosity and anaemia ( Apley’s pg 94 )
* other clinical features include :
- weakness, backache, bone pain, pathological fracture
- hypercalcaemia symptoms (thirst, polyuria and abdominal pain) due to
bone
Resorption
- late 20 features : due to renal dysfunction & spinal cord/root compression
caused by vertebral collapse
D. T –More than half of the patients have Bence-Jones protein in their urine (Apley’s
pg 94)
* investigations :
- mild anaemia
- high ESR
- raised creatinine level + hypercalcaemia
- serum protein electrophoresis : abnormal band
- sterna marrow puncture : plasmacytosis + typical ‘myeloma’ cells
E. T –Specific therapy for multiple myeloma is with alkylating cytotoxic agents -
e.g melphalan ( Apley’s pg 94 )
* treatment :
i. pain control, correct fluid balance & hypercalcaemia and treat pathological #
ii. limb # - internal fixation & packing of cavities with methylmethacrylate cement (
helps to staunch the profuse bleeding ) + antibiotic prophylaxis
iii. spinal # - effective bracing + internal fixation + cord decompression
iv. corticosteroids if bone pain is marked

94. Bone healing

A. T - Healing occurs in three distinct but overlapping stages: 1) the early inflammatory
stage; 2) the repair stage; and 3) the late remodeling stage
(http://www.medscape.com/viewarticle/405699_6)
B. T – Primary cortical healing (direct bone healing) represents an attempt by the
cortex to directly re-establish cortical continuity. This type of healing requires
absolute rigid stabilization (i.e. with a metal plate) after anatomic reduction of the
fracture ends. Regions where the cortical ends are in contact stabilize the other
regions where small gaps are found. Within the gaps, blood vessels will infiltrate
and mesenchymal cells will follow. Osteoclasts at the tip of cutting cones then begin
to bridge the gaps and replace the tiny callus between the bones with new osteons
(“gap healing” process).
(http://www.teambone.com/chapters/basic/fracture.html)
C. T - Secondary fracture healing (indirect bone healing) is a process that relies on the
periosteum for healing where it becomes the primary blood supply to the
surrounding bone. Osteoprogenitor cells within the periosteum are mobilized and
begin to form bone by processes analogous to intramembranous ossification and
endochondral bone formation
(http://www.teambone.com/chapters/basic/fracture.html)
D. F – Healing can occur without callus formation if the fracture site is absolutely
immobile or fracture rigidly immobilized by internal fixation. New bone formation
occurs directly between the fragments. Gaps between fracture surfaces are invaded
by new capillaries & bone forming cells growing in from the edges – “Gap Healing” (
Apley’s pg 269 )
E. F –Remodeling stage can take 3 months to several years to complete
(http://www.teambone.com/chapters/basic/fracture.html)

95. Fracture in children

A. F –Rotational mal-alignment of fracture bone does not remodel.


* these fractures have a low chance of remodeling (may require closed/open
reduction):
- intra articular #
- # with excessive shortening, angulation or rotation
- displaced epiphyseal plate #
- midshaft / diaphyseal #
(Pediatric secrets By Richard Alan Polin, Mark F. Ditmar question no 58 & 59)
B. F– Premature partial or complete closure of growth plate occurs secondary to
epiphyseal #, separated epiphyses and other metaphyseal # closed to growth plates.
Metapyseal alignment avulsion # does not cause growth arrest.
(Pediatric Fractures and Dislocations By Lutz von Laer pg 32)
C. F– Apophysis is connected to bone through a histologically recognizable physis. The
shape and size of apophysis are influenced by the forces placed on it by its muscle or
tendon attachments. Regardless of its location, apophyseal avulsion represents an
epiphysis #. As this avulsion most often occurs in adolescents when the posterior
portion of the growth plate is already closed, growth disturbances are not normally
to be expected
(Pediatric Fractures and Dislocations By Lutz von Laer pg 336)
D. F –Salter Harris Type 3 & 4 demand perfect anatomical reduction ( Apley’s pg 274)
E. T – Preferred treatment for displaced supracondylar humerus fracture is closed
reduction and percutaneous pin fixation using prone-patient positioning where it
can facilitates fracture reduction and safe pin placement while avoiding elbow
hyperflexion
(http://www.wheelessonline.com/ortho/pediatric_supracondylar_fractures_of_th
e_humerus)

96. Definite indication for internal fixation

According to Apley’s pg 286 , indications for internal fixation are :


I. # that cannot be reduced except by operation
II. # that are inherently unstable and prone to re-displacement after reduction
III. # that unite poorly and slowly, principally # of the femoral neck
IV. Pathological # in which bone disease may prevent healing
V. Multiple # where early fixation reduces the risk of general complications
VI. # in pt who present severe nursing difficulties

A. T – (Apley’s pg 286)
B. T – Method of fixation depends on Gustilo’s classification ( if no obvious
contamination, time lapse <8 hours, open fractures of all grades up to IIIa can be
treated as for closed injuries ; cast splintage, intramedullary nailing, plating or
external fixation.
(Apley’s pg 291)
C. T – Internal fixation is indicated in polytrauma to minimize ARDS
(http://www.slideshare.net/abbirr/fracture-management-
w?src=related_normal&rel=2351913)
D. T – (Apley’s pg 286)
E. F – Management of humeral fractures with radial nerve injury remains controversial.
Humeral shaft # with primary radial nerve injury do not usually require nerve
exploration. If the # reduction can be maintained, closed treatment will result in #
healing & good outcome. (Fractures By Donald A. Wiss- pg 69)
F. 97. frozen shoulder (shoulder tendinitis)
G. A. Commonly found in younger age
H. B. Restrict movement in forward flexion only
I. C. Cause by trauma to shoulder
J. D. Self limiting disease
K. E. Recover process is difficult in DM
97. frozen shoulder
A. FALSE
- Commonly found in younger age
- Frozen shoulder should be reserved for a well defined disorder characterized by
progressive pain and stiffness. The patient aged 40-60 may give a history of trauma
often trivial followed by pain. The condition very rarely appears in people under 40.
(apley’s conscise M/S 147, Wikipedia frozen shoulder)

B. FALSE
- Restrict movement in forward flexion only
- Apart from slight wasting, the shoulder looks quite normal, tenderness is seldom
marked. The cardinal feature is a stubborn lack of active and passive movement in all
directions. There is progressive loss of passive ROM (PROM) and active ROM (AROM)
of the glenohumeral joint in a capsular pattern. That is, the movements are usually
restricted to a characteristic pattern, with proportionally greater passive loss of
external rotation than of abduction and internal rotation.
(apleys system of ortho n fractures M/S 287, emedicine frozne shoulder)

C. TRUE
- Cause by trauma to shoulder
- It can be traumatic or idiopathic. Idiopathic disease is more common in older
patients, diabetics and women, other predisposing factor include cervical,
neoplastic, pulmonary, and personality disorders.
(Current essentials orthopedics M/S 53)

D. TRUE
Self limiting disease
- It is an idiopathic disease with 2 principal characteristics: pain and contracture.
- It is usually resolves spontaneously after about 18months but untreated, stiffness
persists for another 6-12months. Gradually movement is regained but may not be
return to nomal. Natural history has 3phases:
i. painful freezing phase (2-9months)
ii. progressive stiffness phase in which motion becomes stiff in all planes while
pain decreases
iii. resolution phase during which range of movement gradually improves
(1month to several years)
(apley’s conscise M/S 147, Current essentials orthopedics M/S 53, emedicine frozen shoulder)

E. Recover process is difficult in DM


- Frozen shoulder in diabetic patients is generally thought to be a more troublesome
condition than in the non-diabetic population, and the recovery is longer.
(source : Wikipedia frozen shouder)

98. Causes of pathological fracture


A. FALSE
- repetitive stress
- A stress or fatigue fracture is one occurring in the normal bone of a healthy patient.
It is caused not by a specific traumatic incident but by repetitive stress, which are of
two main kinds, bending and compression. Most likely occur in new army recruits,
athletes in training and ballet dancers.
(apleys system of ortho n fractures M/S 574)
B. FALSE
Osteoporosis
- Osteoporosis is classify under insufficiency fractures which occur following minimal
trauma to bones that are significantly weaker than normal, typically osteoporotic
and osteomalacic bones.
(apley’s conscise M/S 276)

C. TRUE
Bone cyst
- When abnormal bone gives way, this is referred to as a pathological fracture. The
causes are numerous and varied. Often diagnosis is not made until biopsy is
examined.
- Causes of pathlogical fracture can be classified into 4 categories which are:
i. generalized bone disease (osteogenis imperfect, postmenopausal osteoporosis,
metabolic bone disease, myelomatosis, polyostotic fibrous dysplasia, and
paget’s disease)

ii. local benign conditions (chronic infection, solitary bone cyst, fibrous cortical
defect, chondromyxoid fibroma, aneurismal bone cyst, chondroma, monostotic
fibrous dysplasia)

iii. primary malignant tumours (chondrosarcoma, osteosarcoma, ewing’s tumour)

iv. metastatic tumours (carcinoma from breast, lung, kidney, thyroid, colon, and
prostate)

(apleys system of ortho n fractures M/S 575)

D. FALSE
Previous traumatic fracture
Causes of fractures can be divided into:
i. fractures due to sudden trauma
ii. stress or fatigue fractures
iii. pathological fractures
Previous traumatic fracture is under fractures due to sudden trauma.
(apley’s conscise M/S 265)

E. TRUE
Secondary to bone
This is under 4th categories of the causes of pathological fractures which is metastatic
tumours.
(apleys system of ortho n fractures M/S 575)

99. Anterior shoulder instability

A. FALSE
- Commonly occur in elderly men followed by an acute traumatic event
- Shoulder instability can be divided into anterior instability(95%), posterior instability
and multidirectional instability. For the anterior instability, the patient is usually a
young man who gives a history of his shoulder ‘coming out’ perhaps during a
sporting event. Traumatic anterior instability usually follows an acute injury in which
the arm is forced into abduction, external rotation and extension.
(apleys system of ortho n fractures M/S 289)

B. TRUE
Can cause humeral head articular damage
- Pathology of anterior instability can be either recurrent dislocation or recurrent
subluxation.
- In recurrent dislocation, the labrum and capsule are detached from the anterior rim
of the glenoid ( the classic bankart lesion). In addition, there may be an indentation
on the posterolateral aspect of the humeral head ( the hill-sachs lesion), a
compression fracture due to the humeral head being forced against the anterior
glenoid rim each time it dislocates.
- In recurrent subluxation, the patient may describe a ‘catching’ sensation, followed
by ‘numbness’ or ‘weakness’- dead arm syndrome. Between episodes, the diagnosis
rests on demonstrating the apprehension sign. With the patient seated, the
examiner cautiously lifts the arm into abduction, external rotation, and then
extension, at crucial moment, patient senses humeral head is about to slip out
anteriorly and his body tautens in apprehension.
(apleys system of ortho n fractures M/S 289)

C. TRUE
Is associated with Bankart lesion
From the reasoning in answer (B)

D. TRUE
Positive Apprehension test
From the reasoning in answer (B)

E. TRUE
Mainly treated by conservative treatment
If dislocation recurs only at long intervals, the patient may choose to put up with the
inconvenience.
Indications for operative treatment :
i. frequent dislocations esp if painful
ii. a fear of recurrent subluxation or dislocation
Three types of operation are used:
i. repair or re-attachment of the glenoid labrum (Bankart)
ii. shortening and tightening of the anterior capsule and muscles (Putti-Platt)
iii. reinforcement of the anterior-inferior capsule using adjacent muscles( Bristow)
(apley’s conscise M/S 150)

100. Late complications of fracture include

A. TRUE
Non-union
- Complications of fractures can be divided into early and late.
- Non union is one of the late complications. Minority of cases, delayed union
gradually turns into non-union, and becomes apparent that the fracture will never
unite without intervention.
- Other late complications include:
i. delayed union
ii. malunion
iii. AVN
iv. growth disturbance
v. bed sores
vi. myositis ossificans
vii. tendon lesions
viii. nerve compression
ix. muscle contracture
x. jt instability and stiffness
xi. algodystrophy
xii. OA
(apleys system of ortho n fractures M/S 566)

B. FALSE
Joint contracture
From the reasoning in answer (A)

C. FALSE
Osteomyelitis
- OM is bone infection.
- Infection is one of the early complication of fractures. Open fractures may become
infected, closed fractures hardly ever do unless they are opened by operation. Other
early complication include :
i. visceral injury
ii. vascular injury
iii. nerve injury
iv. compartment syndrome
v. haemarthrosis
vi. gangrene
vii. plaster sores and pressure sores)
(apleys system of ortho n fractures M/S 564)

D. TRUE
Muscle atrophy
- Muscle atrophy is defined as a decrease in the mass of the muscle; it can be a partial
or complete wasting away of muscle.
- Following arterial injury or a compartmental syndrome, the patient may develop
ischaemic contractures of the affected muscles ( volkmann’s ischaemic contracture).
In a severe case affecting the forearm, there will be wasting of the forearm and hand
and clawing of the fingers.
(apleys system of ortho n fractures M/S 572)

E. TRUE
Bleeding
- The fractures most often associated with damage to a major artery. The artery may
be cut, torn, compressed or contused, either by intial injury or subsequently by
jagged bone fragments.
( apleys system of ortho n fractures M/S 562)

You might also like